UPSC-INDIAN-GEO-TEST-2

You might also like

Download as docx, pdf, or txt
Download as docx, pdf, or txt
You are on page 1of 52

1.

Consider the following statements regarding the Archean Rock System in India:
1.These are the oldest rock systems andwere the first to be formed at the timeof cooling and
solidification of theupper crust of the Earth’s surface.
2.The most common Archean rock,covering most of the Peninsula, is thegneiss and schists.
3.They generally have a well-definedfoliated structure.
How many of the above statements is/are correct?
(a)Only one
(b)Only two
(c)All three
(d)None

2.Consider the following statementsregarding the Dharwar Rock System inIndia:


1.This includes the metamorphosedrocks of both sedimentary andigneous origin.
2.Schists, quartzites, slate, limestoneand dolomites are some of the majorrocks of this
system.
3.It is found in the areas of theAravallis, the Himalayas, as well asthe Meghalaya Plateau.
How many of the above statements is/are correct?
(a)Only one
(b)Only two
(c)All three
(d)None

3.Consider the following statementsregarding the Vindhyan Rock System inIndia:


1.It is a vast stratified formation ofsandstones, shales and limestones.
2.This group has a large presence offossil deposits.
3.Large area of this belt is covered bythe Deccan Traps.
How many of the above statements is/are correct?
(a)Only one
(b)Only two
(c)All three
(d)None

4.Consider the following statementsregarding the Gondwana Rock System inIndia:


1.It consists of sandstones with someshales and clays.
2.It is mainly spread in the states ofJharkhand, Chhattisgarh and Odisha.
3.It has abundant deposits of coal, ironore, copper and uranium.
How many of the above statements is/are correct?
(a)Only one
(b)Only two
(c)All three
(d)None

5.Consider the following statements regarding the Deccan Traps in India:


1. They were formed due to the outpourings of extremely mobile basalt lava from fissures and
cracks.
2. They cover parts of Kuchchh, Saurashtra, Maharashtra, the Malwa Plateau, Uttar Pradesh
and northern Karnataka.
3.Its thickness gradually increases fromKuchchh to Amarkantak.
How many of the above statements is/are correct?
(a)Only one
(b)Only two
(c)All three
(d)None

6. Consider the following statements regarding the Trans-Himalayas:


1. It has maximum width in the eastern part.
2. The Krishnagiri Range is found in these Himalayas.
3. The Ladakh Plateau lies to the south-east of the Karakoram Range.
How many of the above statements is/are correct?
(a) Only one
(b) Only two
(c) All three
(d) None

7. Consider the following statements regarding the Punjab Himalayas:


1. Its boundaries are marked by the Indus and the Beas rivers.
2. Zaskar and Pir Panjal Ranges are its part.
3. It is a source of many rivers, like Jhelum.
How many of the above statements is/are correct?
(a) Only one
(b) Only two
(c) All three
(d) None

8. Consider the following statements regarding the Assam Himalayas:


1. They are the eastern section of the Lesser Himalayas, covering Sikkim, Assam and
Arunachal Pradesh.
2. They are bordered by rivers Tista and Brahmaputra.
3. They are the source of many rivers, like Subansiri, Manas, Barak and Sankosh.
How many of the above statements is/are correct?
(a) Only one
(b) Only two
(c) All three
(d) None

9. Consider the following statements regarding the Meghalaya Plateau:


1. It forms a part of the north-eastern hills.
2. It is rich in mineral resources, like coal, iron ore, sillimanite, limestone and uranium.
3. This area receives maximum rainfall from the north-east monsoon.
How many of the above statements is/are correct?
(a) Only one
(b) Only two
(c) All three
(d) None

10. Consider the following statements regarding the Kuchchh Peninsula:


1. It is a source of river Luni.
2. Pachham, Khadir and Bela are found in this part.
3. It lies in the most seismically active zone in India.
How many of the above statements is/are correct?
(a) Only one
(b) Only two
(c) All three
(d) None

11. Consider the following statements regarding the Konkan Plains:


1. They extend from Daman to Goa, covering major portions of Maharashtra.
2. Rivers like Ulhas, Vaitarna, Zuari and Mandovi drain these plains.
Which of the above statements is/are correct?
(a) 1 only
(b) 2 only
(c) Both 1 and 2
(d) Neither 1 nor 2
12. Consider the following statements:
1. The Duncan Passage separates South Andaman and Little Andaman.
2. The Ten Degree Channel separates Minicoy and Maldives.
3. The Eight Degree Channel separates the islands of Minicoy from main Lakshadweep.
4. The Nine Degree Channel separates the Andaman Islands and the Nicobar Islands.
How many of the above statements is/are correct?
(a) Only one
(b) Only two
(c) Only three
(d) All four

13. Consider the following statements:


1. Mt. Saddle is the highest peak of the Andaman and Nicobar Islands, lying in the Great
Nicobar.
2. The Lakshadweep Islands are rich in calcareous deposits.
Which of the statements given above is/ are correct?
(a) 1 only
(b) 2 only
(c) Both 1 and 2
(d) Neither 1 nor 2

14. Consider the following statements:


Statement-I: Bhabar areas are found parallel to the Shiwalik foothills and experience
sudden slope break.
Statement-II: They are characterized by deposition of heavy materials of rocks and boulders.
Which of the following is correct in respect of the above statements?
(a) Both Statement-I and Statement-II are correct and Statement-II is the correct explanation
for Statement-I
(b) Both Statement-I and Statement-II are correct and Statement-II is not the correct
explanation for Statement-I
(c) Statement-I is correct but Statement-II is incorrect
(d) Statement-I is incorrect but Statement-II is correct

15. Consider the following statements regarding the Peninsular Plateau:


1. It is characterized by huge number of Patland Plateaus.
2. The general elevation of the Plateau is from the east to the west.
3. The Central Highlands, because of their unique physiographic features, do not form the
part of the Peninsular Plateau.
How many of the above statements is/are correct?
(a) Only one
(b) Only two
(c) All three
(d) None

16. Consider the following statements regarding the Central Highlands:


1. They slope towards the east and the south-eastern directions.
2. Most of the tributaries of river Yamuna originate from here.
Which of the statements given above is/ are correct?
(a) 1 only
(b) 2 only
(c) Both 1 and 2
(d) Neither 1 nor 2
17. Consider the following statements regarding the Indian Desert:
1. The underlying rock structure of the desert is an extension of the Peninsular Plateau.
2. Playas are the chief characteristic features of this region.
Which of the statements given above is/ are correct?
(a) 1 only
(b) 2 only
(c) Both 1 and 2
(d) Neither 1 nor 2

18. Consider the following pairs:

How many of the above pairs is/are correctly matched?


(a) Only one
(b) Only two
(c) All three
(d) None

19. Consider the following pairs:

How many of the above pairs is/are correctly matched?


(a) Only one
(b) Only two
(c) All three
(d) None

20. With reference to the different drainage patterns, consider the following statements:
1. The drainage pattern, resembling the branches of a tree, is known as ‘dendritic’.
2. When the rivers originate from a hill and flow in all directions, the drainage pattern is
known as ‘centripetal’.
3. When the primary tributaries of the river flow parallel to each other and the secondary
tributaries join them at right angles, the pattern is known as ‘radial’.
How many of the above statements is/are correct?
(a) Only one
(b) Only two
(c) All three
(d) None

21. Arrange the following rivers in north to south direction:


1. Sabarmati
2. Betwa
3. Penganga
4. Krishna
5. Tungabhadra
Select the correct answer using the code given below:
(a) 2 – 1 – 3 – 4 – 5
(b) 1 – 3 – 2 – 5 – 4
(c) 2 – 1 – 5 – 4 – 3
(d) 1 – 2 – 3 – 4 – 5

22. With reference to the Indus river, consider the following statements:
1. It originates from a glacier near Bokhar Chu in the Kailash Mountain Range.
2. The river flows in India only through Jammu and Kashmir.
3. India constructed the Pakal Dul Hydroelectric Project on the Indus river.
How many of the above statements is/are correct?
(a) Only one
(b) Only two
(c) All three
(d) None

23. With reference to the Jhelum river, consider the following statements:
1. It rises from a spring at Verinag, situated in the Pir Panjal Range.
2. It is the largest tributary of the Indus river.
3. The Kishanganga Hydroelectric Project is a run-of-the-river project on the Jhelum river.
How many of the above statements is/are correct?
(a) Only one
(b) Only two
(c) All three
(d) None

24. With reference to the Chenab river, consider the following statements:
1. It is formed by the union of two headwaters, the Chandra and the Bhaga, which rise in
Himachal Pradesh.
2. It was called Asikni in the Rigvedic period.
3. The Marusudar is the biggest tributary of the Chenab.
How many of the above statements is/are correct?
(a) Only one
(b) Only two
(c) All three
(d) None

25. With reference to the Himalayan rivers, consider the following statements:
1. The Ravi river rises from the Rohtang Pass in the Kullu hills.
2. The Beas river flows through the states of Punjab and Himachal Pradesh.
3. Both the Ravi and the Beas rivers are transboundary rivers.
How many of the above statements is/are correct?
(a) Only one
(b) Only two
(c) All three
(d) None

26. With reference to the Sutlej river, consider the following statements:
1. The Sutlej river is an antecedent river.
2. The river originates from the ‘Raksas Tal’ near the Mansarovar lake.
3. It meets river Ravi near Harike Pattan in Punjab.
How many of the above statements is/are correct?
(a) Only one
(b) Only two
(c) All three
(d) None

27. With reference to the Ganga river, consider the following statements:
1. It rises in the Gangotri glacier near Gaumukh, where it is known as the Bhagirathi.
2. At Rudra Prayag, the Bhagirathi meets the Alaknanda to form the Ganga.
Which of the above statements is/are correct?
(a) 1 only
(b) 2 only
(c) Both 1 and 2
(d) Neither 1 nor 2

28. With reference to the Yamuna river, consider the following statements:
1. It originates from the Yamunotri glacier on the eastern slopes on the Kailash Range.
2. It is the western-most and the longest tributary of river Ganga.
3. The Hindan, the Rind, the Ken and the Betwa are its major tributaries.
How many of the above statements is/are correct?
(a) Only one
(b) Only two
(c) All three
(d) None

29.With reference to the Himalayan rivers,consider the following statements:


1.River Gandak rises from the NepalHimalayas and joins the Ganga atSonpur near Patna.
2.The Ghaghara originates in theglaciers of Mapchachungo.
3.Both the Gandak and the Ghagharaare the right bank tributaries of theGanga river.
How many of the above statements is/are correct?
(a)Only one
(b)Only two
(c)All three
(d)None

30.Consider the following statements:


1.At Tarai belt, most of the streams andrivers re-emerge without having anyproperly
demarcated channel.
2.Tarai has luxurious growth of naturalvegetation.
Which of the statements given above is/are correct?
(a)1 only
(b)2 only
(c)Both 1 and 2
(d)Neither 1 nor 2
31.Consider the following statements:
1.The Northern Plains are formed by thealluvial deposits brought by the riversGanga and
Indus only.
2.The Northern Plains are one of theoldest and the most stable landmassof India.
3.Haryana and Delhi form a waterdivide between the Indus and theGanga river systems.
How many of the above statements is/are correct?
(a)Only one
(b)Only two
(c)All three
(d)None

32.Consider the following statements:


1.Tors, block mountains, rift valleys,spurs and hummocky hills arecommonly found in
PeninsularPlateau.
2.The Karbi-Anglong Plateau is anextension of the Peninsular Plateau.
Which of the above statements is/are correct?
(a)1 only
(b)2 only
(c)Both 1 and 2
(d)Neither 1 nor 2

33.Consider the following statements:


1.The Western Ghats are locally knownas Sahyadri in Maharashtra.
2.Dodabetta is the highest peak of thePeninsular Plateau, located in theWestern Ghats.
3.The Eastern and the Western Ghatsmeet each other at the Nilgiri Hills.
How many of the above statements is/are correct?
(a)Only one
(b)Only two
(c)All three
(d)None

34.Arrange the following Himalayan peaks ineast to west direction:


1.Nanga Parbat
2.Kamet
3.Namcha Barwa
4.Kanchenjunga
Select the correct answer using the code given below:
(a)3 – 2 – 4 – 1
(b)1 – 2 – 4 – 3
(c)3 – 4 – 2 – 1
(d)2 – 3 – 4 – 1

35.Arrange the following hills in west to eastdirection:


1.The Satmala Hills
2.The Garhjat Hills
3.The Rajmahal Hills
4.The Kaimur Hills
Select the correct answer using the code given below:
(a) 1 – 4 – 2 – 3
(b) 2 – 3 – 4 – 1
(c) 1 – 4 – 3 – 2
(d) 3 – 2 – 4 – 1
36. Consider the following pairs:

How many of the


above pairs is/are correctly matched?
(a) Only one
(b) Only two
(c) All three
(d) None

37. Consider the following states:


1. Odisha
2. Jharkhand
3. West Bengal
4. Rajasthan
5. Madhya Pradesh
The Tropic of Cancer passes through how many of the above states?
(a) Only two
(b) Only three
(c) Only four
(d) All five

38. Consider the following statements:


1. The Kosi is an antecedent river.
2. The Ramganga river rises in the Mukteshwar Hills in the Kumaon Himalayas.
3. The Damodar and the Barakar rivers trifurcate the Chotanagpur Plateau.
How many of the above statements is/are correct?
(a) Only one
(b) Only two
(c) All three
(d) None

39. Consider the following statements:


1. The Saryu river rises in the Milam Glacier.
2. The Mahananda river joins the Ganga as its last left bank tributary in West Bengal.
3. The Son originates in the Amarkantak Plateau.
How many of the above statements is/are correct?
(a) Only one
(b) Only two
(c) All three
(d) None
40. With respect to the “Brahmaputra” river, consider the following statements:
1. It has its origin in the Chemayungdung Glacier of the Kailash Range near the Mansarovar
Lake.
2. The major tributaries are the Kameng, the Subansiri and the Sankosh.
3. It enters India near Sadiya town in Arunachal Pradesh.
How many of the above statements is/are correct?
(a) Only one
(b) Only two
(c) All three
(d) None

41. Which of the following geological events in the distant past has/have shaped the present
drainage systems of the Peninsular India?
1. Subsidence of the eastern flank of the Peninsula.

2. Upheaval of the Himalayas.


3. Slight tilting of the Peninsular block from the north-east to the south-western direction.
Select the correct answer using the code given below:
(a) 1 only
(b) 2 only
(c) 2 and 3 only
(d) 1,2 and 3

42. With respect to the “Peninsular Drainage System”, consider the following statements:
1. This system is characterised by fixed course, absence of meanders and non-perennial flow
of water.
2. This drainage system is characterised by the deep gorges, V-shaped valleys and rapids.
Which of the statements given above is/are correct?
(a) 1 only
(b) 2 only
(c) Both 1 and 2
(d) Neither 1 nor 2

43. With respect to the “Godavari” river, consider the following statements:
1. It is the second largest Peninsular river system.
2. It rises in the Nasik district of Maharashtra.
3. Its tributaries run through the states of Maharashtra, Madhya Pradesh, Chhattisgarh,
Odisha and Andhra Pradesh.
4. The Penganga, the Bhavani, the Pranhita and the Manjra are its principal tributaries.
How many of the above statements is/are correct?
(a) Only one
(b) Only two
(c) Only three
(d) All four

44. Consider the following statements:


1. The Kaveri rises in the Kudremukh Hills in Karnataka.
2. The Krishna is an east-flowing Peninsular river, which rises near Mahabaleshwar in
Sahyadri.
3. The Koyna, the Tungbhadra and the Bhima are the major tributaries of the Kaveri.
How many of the above statements is/are correct?
(a) Only one
(b) Only two
(c) All three
(d) None
45. Consider the following statements:
1. The Narmada river forms estuaries.
2. The Narmada is one of the rivers in India that flows in a rift valley, bordered by the
Satpura and the Vindhyan Ranges.
3. The Tapi is a westward flowing river.
How many of the above statements is/are correct?
(a) Only one
(b) Only two
(c) All three
(d) None

46. Consider the following statements:


1. The Mahanadi rises near Sihawa in Raipur district of Chhattisgarh.
2. Luni is the largest river system of Rajasthan.
3. The drainage basin of the Mahanadi river lies in Madhya Pradesh, Chhattisgarh, Odisha
and West Bengal.
How many of the above statements is/are correct?
(a) Only one
(b) Only two
(c) All three
(d) None

47. The Kanwar lake, Asia’s largest freshwater oxbow lake, is a residual part of which of the
following rivers?
(a) Brahmaputra river
(b) Godavari river
(c) Son river
(d) Gandak river

48. Consider the following statements:


1. Biligirirangan Hills is located at the north-west of the Western Ghats and westernmost
edge of the Eastern Ghats.
2. Tributaries of the Penneru River drain the Seshachalam Hills.
Which of the statements given above is/are correct?
(a) 1 only
(b) 2 only
(c) Both 1 and 2
(d) Neither 1 nor 2

49. Consider the following statements:


1. The volcanic, clay-like soil of the Malwa plateau owes its black colour to the high iron
content of the basalt from which it formed.
2. Sigar and Janapav peaks are located in the Malwa plateau.
Which of the statements given above is/are correct?
(a) 1 only
(b) 2 only
(c) Both 1 and 2
(d) Neither 1 nor 2
50. Consider the following pairs:

How many pairs given above is/are correctly matched?


(a) Only one
(b) Only two
(c) All three
(d) None

51. Consider the following statements regarding the weather conditions over India during the
winter months:
1. A low-pressure centre in the region lying to the north of the Himalayas develops during
winter.
2. North India experiences north-westerly dry cold winds.
3. North-easterly jet stream brings cyclonic disturbances from the Mediterranean region
towards India.
How many of the above statements is/are correct?
(a) Only one
(b) Only two
(c) All three
(d) None

52. Consider the following statements regarding the weather conditions over India during the
summer months:
1. The maritime tropical airmass (mT) from the southern hemisphere rushes towards India.
2. Easterly jet stream flows over the southern part of the Peninsula.
3. The Inter Tropical Convergence Zone shifts northwards towards the Gangetic plains.

How many of the above statements is/are correct?


(a) Only one
(b) Only two
(c) All three
(d) None

53. Consider the following statements regarding the Indian monsoon:


1. The onset of monsoon is first experienced by Kerala in India.
2. Much of the rainfall along the Western Ghats is convectional.
3. Parallel blowing of winds along the west coast leads to break in monsoon in those areas.
How many of the above statements is/are correct?
(a) Only one
(b) Only two
(c) All three
(d) None
54. Consider the following statements:
1. The Peninsular region of India has a well-defined cold weather season.
2. During the winters, the weather in India is generally pleasant.
3. Tamil Nadu coast and southern Andhra Pradesh receive most of their rainfall from the
north-east monsoon.

How many of the above statements is/are correct?


(a) Only one
(b) Only two
(c) All three
(d) None

55. Consider the following statements regarding the retreating monsoon:


1. It is marked by clear skies and fall in temperature.
2. The weather in the retreating monsoon is dry in north India.

Which of the statements given above is/are correct?


(a) 1 only
(b) 2 only
(c) Both 1 and 2
(d) Neither 1 nor 2

56. Consider the following factors:


1. Latitude
2. Himalayan mountains
3. Distribution of land and water
4. Distance from the sea
5. Altitude
6. Relief
Which of the above factors influence the Indian climate?
(a) 1, 2, 3 and 5 only
(b) 1, 2, 4 and 5 only
(c) 1, 2, 4, 5 and 6 only
(d) 1, 2, 3, 4, 5 and 6

57. Consider the following statements regarding El Nino:


1. Warm ocean water appears near the coast of Peru.
2. Trade winds get weaken across the Pacific.
3. India always experiences droughts.

How many of the above statements is/are correct?


(a) Only one
(b) Only two
(c) All three
(d) None

58. Consider the following statements regarding the Madden-Julian Oscillation:


1. It is an eastward moving 'pulse' of clouds, rainfall and winds near the Equator.
2. When it is over the Indian Ocean during the monsoon season, it brings good rainfall over
the Indian sub-continent.

Which of the statements given above is/are correct?


(a) 1 only
(b) 2 only
(c) Both 1 and 2
(d) Neither 1 nor 2
59. Consider the following statements regarding cloudburst:
1. It refers to an extremely heavy downpour over a large area for a long duration.
2. Upward moving air currents do not allow the rain-drops to fall on the ground and thus,
accumulate them at higher levels.

Which of the statements given above is/are correct?


(a) 1 only
(b) 2 only
(c) Both 1 and 2
(d) Neither 1 nor 2

60. Consider the following factors:


1. Earthquakes
2. Rainfall
3. Changes in ground water
4. Mining
5. Nuclear tests
6. Dam building

Which of the above factors can cause landslides?


(a) 1, 2, 3 and 5 only
(b) 1, 2, 4 and 5 only
(c) 1, 2, 4, 5 and 6 only
(d) 1, 2, 3, 4, 5 and 6

61. Consider the following statements:


1. The Circum-Pacific Belt is characterized by active volcanoes, earthquakes and landslides.
2. The Indus-Tsangpo Suture Zone is characterized by active volcanoes, earthquakes and
landslides.

Which of the statements given above is/are correct?


(a) 1 only
(b) 2 only
(c) Both 1 and 2
(d) Neither 1 nor 2

62. Which of the following natural events is characterized by the rapid and massive
displacement of water, often caused by undersea geological activities, resulting in the
generation of powerful oceanic waves?
(a) Cyclone
(b) Tornado
(c) Tsunami
(d) Blizzard

63. Which of the following statements is/are the correct explanation(s) of Tamil Nadu
remaining dry during summers in India?
1. The Tamil Nadu coast is situated parallel to the Bay of Bengal branch of the south-west
monsoon.
2. It lies in the rain shadow area of the Arabian Sea branch of the south-west monsoon.

Select the correct the answer using the code given below:
(a) 1 only
(b) 2 only
(c) Both 1 and 2
(d) Neither 1 nor 2
64. Consider the following statements:
1. Nor Westers are hot, dry and oppressing winds blowing in the northern plains from Punjab
to Bihar.
2. Loo is also known as ‘Kalbaisakhi’.
3. Bardoli Chheerha in Assam is beneficial for tea, rice and jute.
4. Blossom Showers are beneficial for coffee production in the state of Kerala.
How many of the above statements is/are correct?
(a) Only one
(b) Only two
(c) Only three
(d) All four

65. With reference to the tropical evergreen forests in India, consider the following
statements:
1. These forests are found in the eastern slope of the Western Ghats.
2. They are found in warm and humid areas with an annual precipitation of over 200 cm.
3. There is no definite time for the trees to shed their leaves, flowering and fruition.
How many of the above statements is/are correct?
(a) Only one
(b) Only two
(c) All three
(d) None

66. In India, in which one of the following types of forests is the white cedar a dominant tree
species?
(a) Tropical moist deciduous forests
(b) Tropical rainforests
(c) Tropical semi-evergreen forests
(d) Temperate forests with grasslands

67. With reference to the tropical thorn forests, consider the following statements:
1. In these forests, the plants remain leafless for most part of the year.
2. They occur in the semi-arid areas of south-west Punjab, Haryana and Gujarat.
Which of the statements given above is/are correct?
(a) 1 only
(b) 2 only
(c) Both 1 and 2
(d) Neither 1 nor 2

68. Consider the following statements:


1. Deciduous forests are found in the foothills of the Himalayas.
2. The northern slopes of the Himalayas carry a thicker vegetation than the southern slopes.
3. The southern mountains are devoid of temperate forests.
How many of the above statements is/are correct?
(a) Only one
(b) Only two
(c) All three
(d) None

69. Consider the following statements:


1. Moist deciduous forests are found in the north-eastern states along the foothills of the
Himalayas.
2. Tropical deciduous forests are the most widespread forests in India.
3. Teak, sal and sandalwood are the main species of dry deciduous forests.
How many of the above statements is/are correct?
(a) Only one
(b) Only two
(c) All three
(d) None

70. With reference to the dry deciduous forests, consider the following statements:
1. These forests have a parkland landscape in the higher rainfall regions of the Peninsular
Plateau.
2. Tendu, amaltas and khair are the common trees of these forests.
Which of the statements given above is/are correct?
(a) 1 only
(b) 2 only
(c) Both 1 and 2
(d) Neither 1 nor 2

71. Consider the following statements:


1. In India, Nanda Devi was the first Protected Area to be designated as a Biosphere Reserve
under the UNESCO’s Man and the Biosphere Programme.
2. The Similipal Biosphere Reserve is a part of Dibrugarh and Tinsukia districts in Assam.
3. The Nokrek Biosphere Reserve is a part of East, West and South Garo Hill districts in
Meghalaya.
How many of the above statements is/are correct?
(a) Only one
(b) Only two
(c) All three
(d) None

72. Consider the following statements about mangroves:


1. They are characterized by halophytic trees, shrubs and other plants growing in brackish to
saline tidal waters.
2. Mangrove species have specialized above-ground roots, called the pneumatophores.
3. The Baratang Island mangrove is located in Lakshadweep.
How many of the above statements is/are correct?
(a) Only one
(b) Only two
(c) All three
(d) None

73. With reference to wheat cultivation, consider the following statements:


1. Soils with a loam texture and moderate water holding capacity are ideal for wheat
cultivation.
2. Cool and moist weather enables the wheat grains to ripen properly.
3. Wheat crops can be grown in the temperate zones.
How many of the above statements is/are incorrect?
(a) Only one
(b) Only two
(c) All three
(d) None

74. With reference to millets, consider the following statements:


1. Ragi is a crop of dry regions and grows well only on alluvial soils.
2. Jowar is a rain-fed crop mostly grown in the moist areas which hardly need irrigation.
Which of the statements given above is/are correct?
(a) 1 only
(b) 2 only
(c) Both 1 and 2
(d) Neither 1 nor 2
75. With reference to the cultivation of maize, consider the following statements:
1. It requires temperature between 21°C and 27°C, and grows well in old alluvial soil.
2. Maize is grown as a kharif crop throughout the country.
3. Soils with good organic matter content, having high water holding capacity with neutral
pH, are good for higher productivity.
How many of the above statements is/are correct?
(a) Only one
(b) Only two
(c) All three
(d) None

76. With reference to the cultivation of pulses, consider the following statements:
1. Being leguminous crops, all pulses help in restoring soil fertility by fixing nitrogen from the
air.
2. Arhar is grown in the rabi season, while moong is a kharif crop.
Which of the statements given above is/are correct?
(a) 1 only
(b) 2 only
(c) Both 1 and 2
(d) Neither 1 nor 2

77. With reference to sugarcane, consider the following statements:


1. Sugar recovery is the highest when the weather is dry with low humidity.
2. Yield levels of sugarcane are higher in the southern states, than northern India.
Which of the statements given above is/are correct?
(a) 1 only
(b) 2 only
(c) Both 1 and 2
(d) Neither 1 nor 2

78. With reference to oilseeds, consider the following statements:


1. Majority of the oilseeds are cultivated under the rain-fed ecosystem.
2. Groundnut accounts for about half of the major oilseeds produced in the country.
3. Sesamum and castor seeds are grown both as rabi and kharif crops.
How many of the above statements is/are correct?
(a) Only one
(b) Only two
(c) All three
(d) None

79. With reference to the production of tea in India, consider the following statements:
1. The tea plant grows well in a well-drained soil which is devoid of humus and organic
matter.
2. Tea bushes require a warm and moist frost-free climate all throughout the year.
3. Munnar in Karnataka is one of the leading tea producing regions in India.
How many of the above statements is/are correct?
(a) Only one
(b) Only two
(c) All three
(d) None

80. “The crop is an equatorial crop, which requires a moist and humid climate with rainfall of
more than 200 cm., and temperature above 25°C. It acts as an important industrial raw
material and is mainly grown in Kerala, Tamil Nadu, Karnataka and the Andaman & Nicobar
Islands and the Garo hills of Meghalaya.” Which one of the following is this crop?
(a) Jute
(b) Cashewnut
(c) Rubber
(d) Black Pepper

81. With reference to cotton cultivation in India, consider the following statements:
1. Cotton is a crop grown in the rabi season.
2. Cotton is semi-tolerant to salinity and sensitive to water logging and thus, prefers well-
drained soils.
3. Cotton is grown in the tropical and sub-tropical conditions.
How many of the above statements is/are correct?
(a) Only one
(b) Only two
(c) All three
(d) None

82. Consider the following statements:


1. The Bhakra Nangal Dam is a joint venture between two states.
2. The Nangal Dam has created a reservoir, known as the Gobind Sagar Lake.
3. The Bhakra Dam is a gravity dam, situated in the state of Haryana.
4. The Bhakra Nangal Dam is home to the endangered fish species 'Mahseer'.
How many of the above statements is/are correct?
(a) Only one
(b) Only two
(c) Only three
(d) All four

83. With reference to the Damodar Valley Project in India, consider the following statements:
1. The Damodar Valley Project is the first multi-purpose river valley project of independent
India.
2. The Konar Dam has been constructed over the Barakar river.
3. The command area of the project includes the districts of Bihar, Jharkhand and West
Bengal.
How many of the above statements is/are incorrect?
(a) Only one
(b) Only two
(c) All three
(d) None

84. Regarding the Hirakud Dam Project in India, consider the following statements:
1. The dam is built across the Mahanadi river in the state of Odisha.
2. The reservoir created by the dam has been declared as a Ramsar Site.
3. The Debrigarh Wildlife Sanctuary is situated near the Hirakud Reservoir.
How many of the above statements is/are correct?
(a) Only one
(b) Only two
(c) All three
(d) None

85. With reference to the Chambal river, consider the following statements:
1. The river originates from Mhow, situated in the Aravalli Range.
2. The Banas is a major tributary originating in the Vindhyan Range.
3. The Gandhi Sagar Reservoir has been constructed for irrigation purposes on the Chambal
river.
How many of the above statements is/are correct?
(a) Only one
(b) Only two
(c) All three
(d) None
86. With reference to the multi-purpose projects in India, consider the following statements:
1. The reservoir Gobind Ballabh Pant Sagar, formed by the Rihand Dam, is the largest
artificial lake in India.

2. The catchment area of the Rihand Dam extends over Uttar Pradesh, Madhya Pradesh and
Chhattisgarh.
Which of the statements given above is/are correct?
(a) 1 only
(b) 2 only
(c) Both 1 and 2
(d) Neither 1 nor 2

87.
Consider
the
following Multi-
pairs: purpose Location
Project
S.N.
1. Sardar : Gujarat
Sarovar
Project
2. Gandak : Uttar Pradesh
Project
3. Idukki : Kerala
Project
4. Bheema : Karnataka
Project

How many of the above pairs is/are correctly matched?


(a) Only one
(b) Only two
(c) Only three
(d) All four

88. With reference to the Nagarjuna Sagar Project in India, consider the following statements:
1. The Nagarjuna Sagar Project is constructed on the Indravati river.
2. The dam has led to the shrinkage of the Kolleru Lake.
Which of the statements given above is/are correct?
(a) 1 only
(b) 2 only
(c) Both 1 and 2
(d) Neither 1 nor 2

89. Consider the following statements:


1. The Tehri Dam has been constructed at the confluence of the Bhilganga and the
Bhagirathi rivers.
2. The Koteshwar Dam forms the lower reservoir of the Tehri Dam.
3. The Tehri Dam is located within the geological fault region, the Central Himalayan Seismic
Gap.
How many of the above statements is/are incorrect?
(a) Only one
(b) Only two
(c) All three
(d) None

90. With reference to the Beas Project in India, consider the following statements:
1. The Maharana Pratap Bird Sanctuary, created by the dam, is situated in the state of
Himachal Pradesh.
2. The Pong Dam, under the Beas Project, is situated in the Dhaoladhar Range.
3. The project is a joint venture between the states of Punjab, Himachal Pradesh and
Haryana.
How many of the above statements is/are correct?
(a) Only one
(b) Only two
(c) All three
(d) None

91. With reference to the alluvial soils, consider the following statements:
1. It is a depositional soil, transported and deposited by the rivers and the streams.
2. It is generally rich in phosphorous, but poor in potash.
3. The alluvial soils vary in nature from sandy loam to clay.
How many of the above statements is/are correct?
(a) Only one
(b) Only two
(c) All three
(d) None

92. With reference to the black soils, consider the following statements:
1. It is an in-situ soil that covers most of the Deccan Plateau.
2. It is generally clayey, deep and self-ploughing soil.
3. It is rich in phosphorous, nitrogen and organic matter.
How many of the above statements is/are correct?
(a) Only one
(b) Only two
(c) All three
(d) None

93. With reference to the red and yellow soils, consider the following statements:
1. Red soil develops on crystalline igneous rocks in the areas of low rainfall of the Deccan
Plateau.
2. The soil develops a reddish colour due to a wide diffusion of iron in crystalline and
metamorphic rocks.
3. The fine-grained red and yellow soils are normally fertile.
How many of the above statements is/are incorrect?
(a) Only one
(b) Only two
(c) All three
(d) None

94. With reference to the laterite soil, consider the following statements:
1. It develops in the areas with low temperature and low rainfall.
2. It is more suitable for tree crops, like cashewnut.
3. These soils are poor in organic matter and nitrogen.
How many of the above statements is/are correct?
(a) Only one
(b) Only two
(c) All three
(d) None
95. ‘The soil ranges from red to brown in colour, sandy in structure and saline in nature.
Nitrogen is insufficient and the phosphate content is normal in this soil. These soils are poor,
and contain little humus and organic matter.’
The above statements reflect which of the following soils of India?
(a) Peaty soils
(b) Saline soils
(c) Arid soils
(d) Laterite soils

96. With reference to the saline soil, consider the following statements:
1. It is an infertile soil and does not support any vegetative growth.
2. Seawater intrusions in the deltas promote the occurrence of this soil.
3. Gypsum is added to solve the problem of salinity in the soil.
How many of the above statements is/are correct?
(a) Only one
(b) Only two
(c) All three
(d) None

97. With reference to the peaty soil, consider the following statements:
1. It is found in the areas of high humidity.
2. It occurs in the coastal areas of West Bengal, Odisha and Tamil Nadu.
3. It is normally heavy and black in colour.
How many of the above statements is/are correct?
(a) Only one
(b) Only two
(c) All three
(d) None

98. With reference to the forest soil, consider the following statements:
1. It is loamy and silty on the upper slopes and coarse-grained on the valley sides.
2. In the snow-bound areas of the Himalayas, it experiences denudation.
3. The soils found in the upper slopes are fertile.
How many of the above statements is/are correct?
(a) Only one
(b) Only two
(c) All three
(d) None

99. Consider the following statements:


1. Sheet erosion occurs on the level lands.
2. Gully erosion is common on steep slopes.
3. A region with a large number of steep erosions is called a badland topography.
How many of the above statements is/are correct?
(a) Only one
(b) Only two
(c) All three
(d) None

100. Consider the following statements:


1. The Kamarajar Port/Ennore port is located in the Coromandel Coast in the Bay of Bengal.
2. Paradip Port is at the confluence of the Mahanadi river and the Bay of Bengal.
Which of the statements given above is/are correct?
(a) 1 only
(b) 2 only
(c) Both 1 and 2
(d) Neither 1 nor 2
ANSWERS AND EXPLANATIONS

1. Answer: (c)
Explanation:
The Archaean System: Gneisses and schists are the oldest rocks and were the first to be
formed at the time of cooling and solidification of the upper crust of the Earth's surface in the
pre-Cambrian era (about 4,000 million years ago). They are thoroughly crystalline, extremely
contorted and faulted, often formed as plutonic intrusions and generally have a well-defined
foliated structure. They often underlie the strata formed subsequently and the system is
generally known by names of the ‘Fundamental Complex' or the 'Basement Complex'.
The most common Archaean rock, covering about two-thirds of the Peninsular surface, is the
gneiss. This is the rock which, in mineral composition, may vary from granite to gabro,
possesses a constant, more or less foliated or banded structure. The schists, mostly
crystalline, include mica, tale, hornblende, chlorite, epidote sillimanite and graphite schists.
The crystalline metamorphosed sediments and gneissic rocks cover large parts of India. The
central and the southern parts of the Peninsula are occupied by this rock system.
To the north-east of the Peninsula, they occupy wide areas in Odisha, Meghalaya, Madhya
Pradesh, Chhattisgarh and the Chotanagpur Plateau of Jharkhand.
They also cover the whole of Bundelkhand in the north and to the north-west, they are found
in a number of isolated outcrops, extending from north of Vadodara to a long distance along
the Aravallis.
In the extra-peninsula, these rocks are exposed all along the Himalayas, forming the bulk of
the high ranges and the backbone of the mountain system.

2. Answer. (c)
Explanation:
The Dharwar System derives its name from the rocks first studied in the Dharwar district of
Karnataka, where such rocks are found in abundance. The Dharwars include some of the
highly metamorphosed rocks of both sedimentary and igneous origin.
The Dharwar System is the most ancient metamorphosed sedimentary rock-system of India,
as old as, and in some cases older than, the basement gneisses and schists.
The weathering of the pristine Archaean gneisses and schists yielded the earliest sediments,
which were deposited on the bed of the sea, and formed the oldest sedimentary strata, known
in the geology of India as the Dharwar System.
The major rocks of the Dharwar System are hornblende, schists, quartzites, phyllites, slates,
crystalline limestones and dolomites.
The Dharwar System is very well developed in the Dharwar-Bellary-Mysore belt of Karnataka.
It also occurs in Jharkhand (Ranchi, Hazaribagh); Madhya Pradesh (Balaghat, Rewa);
Chhattisgarh (Bastar, Dantewara, Kanker); Odisha (Sundergarh, Keonjhar); and in the
Aravallis, between Jaipur and Palanpur.
In the extra-Peninsular region, the Dharwar System is well represented in the Himalayas,
both in the central and the northern zones, as well as in the Meghalaya Plateau.
The Dharwars are economically the most important rocks, because they possess
valuable minerals, like high grade iron-ore, manganese, copper, lead, gold, quartzites, slates,
mica, etc.

3. Answer: (b)
Explanation:
The Vindhyan System (1,300-600 million years): This System derives its name from the
Great Vindhyan Mountains.
The System comprises ancient sedimentary rocks, super-imposed on the Archaean base. It is
a vast stratified formation of sandstones, shales and limestones, often over 4,000 m thick.
Except a few traces of animal and vegetable life, this group is devoid of any
recognizable fossils.
Occupying a large area of over 1,00,000 sq km., the Vindhyan System stretches from
Sasaram and Rohtas in western Bihar to Chittaurgarh in Rajasthan, with the exception of a
central tract in Bundelkhand, which makes a gap in this belt.
Large area of this belt is covered by the Deccan Trap. These rocks are also found in
Chhattisgarh, the Bhima valley of Karnataka and Kurnool district of Andhra Pradesh.
The Vindhyan System has been found to continue to the north under the Gangetic alluvium
and they are perhaps buckled down underneath the Himalaya.
The Lower Vindhyan (1,300-1,100 million years) is marine in origin, mostly calcareous in
nature and shows tectonic deformation by folding movements.
This System is well placed in the Son valley, in Chhattisgarh, in the valley of the Bhima and
in a separate basin in Mewar.
The Upper Vindhyan beds enclose two diamond bearing horizons, from which Panna and
Golconda diamonds have been mined.
The Vindhyan System, on the whole, is devoid of metalliferous minerals, but provides large
quantities of excellent and durable free stones, flagstones, ornamental stones, limestone,
pure glass making sand and some coal.

4. Answer. (c)
Explanation:
The Gondwana System derives its name from the kingdom of the Gonds, the most primitive
people living in Andhra Pradesh. It has relevance with the Gond region of Madhya Pradesh
also, where these rocks were first discovered.
The Gondwanas consist of sandstones with some shales and clays. They are of continental
origin, fluviatile and lacustrine deposits laid down in geo-synclinal troughs on ancient
plateau surface. As the sediments accumulated, the loaded troughs subsided, which led to
thick deposits of fresh water and sub-aerial sediments into which were embedded the
terrestrial plants and animals.
The main areas of Gondwana rocks in the Peninsula are along the Damodar valley in
Jharkhand, along the Mahanadi river valley in Chhattisgarh and Odisha, in the southern
part of Madhya Pradesh and a series of troughs along the Godavari from Nagpur to the delta.
In the extra peninsular region, these rocks are found in Kashmir, Darjeeling and Sikkim.
Here, they are so much deformed, that they have lost their original identity and are entirely
different from the peninsular rocks.
Economically, the Gondwana rocks are the most important in India, containing about 98 per
cent of her coal reserves. They have rich deposits of iron ore, copper, uranium and antimony.
Sandstones, slates and conglomerates are used as building materials.

5. Answer: (b)
Explanation:
The Deccan Trap: From the end of the Cretaceous, till the beginning of the Eocene,
stupendous volcanic outburst overwhelmed a vast area of the Peninsular India, like the one
which is seldom known anywhere else in the world geological history. A vast area of about ten
lakh sq km., was flooded by the outpourings of extremely mobile basalt lava from fissures
and cracks covering fully the pre-existing topography. These volcanic deposits have flat top
and steep sides, so that they appear as gigantic steps from a distance, and therefore, called
‘Trap’, the name derived from the Swedish word meaning a 'stair’ or 'step'.
The process of denudation over a long period has reduced the Deccan Trap to almost half of
its original size and the present Deccan Trap covers about 5 lakh sq km., mainly in parts of
Kuchchh, Saurashtra, Maharashtra, the Malwa Plateau and northern Karnataka. Parts of
Andhra, Tamil Nadu, Jharkhand and Uttar Pradesh also have some outliers of the Deccan
Trap. The thickness of the Deccan Traps is not uniform everywhere.
It is as much as 3,000 metres along the coast of Mumbai, which is reduced to 600-800
metres towards the southern limit, 800 metres in Kuchchh and only 150 metres at
Amarkantak, near its eastern limit.
The Deccan Trap has been divided into the following groups:
Basalt is the main rock, but dolorite, rhyolite, gabro and many other rocks are also found.
These rocks are a great source of quartz, agate, calcite, building stones and road building
material. The weathering of these rocks, for a long time, has given birth to the black cotton
soil, known as 'regur'.

6. Answer: (a)
Explanation:
The Trans-Himalayas: This part of the Himalayan Ranges is also called the Tibetan
Himalaya, because most of it lies in Tibet.
The Zaskar, the Ladakh, the Kailas and the Karakoram are the main ranges of the Trans-
Himalayan system. It stretches for a distance of about 1,000 km in east-west direction and
its average elevation is 3,000 m above the mean sea level. The average width of this region
is 40 km at the eastern and western extremities, and about 225 km in the central part.
The Zaskar Range branches off from the Great Himalayan Range near 80" E longitude, and
runs more or less parallel to it. The Nanga Parbat (8,126 m) forms its culmination in the
north-west, but the adjoining Deosai mountain may also be included in it. North of the
Zaskar Range and running parallel to it is the Ladakh Range. It is about 300 km long and its
average elevation is 5,800 metre above the sea level. Only a few peaks of this Range attain
heights of over 6,000 metres. The Rakaposhi-Haramosh Ranges beyond the Indus may be
treated as the extensions of the Ladakh Range to the north-west.
The Kailas Range (Gangdise in Chinese) in western Tibet is an offshoot of the Ladakh Range.
Its average elevation is 5,500-6,000 m above the sea level and its average width is 30 km. The
highest peak is Mount Kailas (6,714 m). River Indus originates from the northern slopes of
the Kailas Range.
The northern most range of the Trans-Himalayan Ranges in India is the Great Karakoram
Range, also known as the Krishnagiri Range. It forms India's frontier with Afghanistan and
China. It is the abode of some of the greatest glaciers of the world outside the Polar regions.
K2 (8,611 m) is the second highest peak in the world and the highest peak in the Indian
Union (excluding the auxiliary peaks of the Great Himalayas) and rises majestically like a
cone. It has been named as Godwin Austen by the British and Qogir by the Chinese.
The Ladakh Plateau lies to the north-east of the Karakoram Range. With an average
elevation of over five thousand metres above the sea level, it is the highest plateau of the
Indian Union.

7.Answer: (b)
Explanation:
The Western Himalayas, also called thePunjab Himalayas, are the western-mostsection of the
vast Himalayas MountainRange. It lies mainly in the Kashmirregion of the northern Indian
sub-continent and also in the north-westernpart of Himachal Pradesh state, India.
In all, the Western Himalayas extendsouth-east for some 350 miles (560km) from the
bend of the Indus river(north-west) to the Sutlej river (south-east). The Upper Indus
separates themfrom the Karakoram Range to the north.
Included within the Western Himalayasare the Zaskar Range, the Pir PanjalRange, and parts
of the Siwalik Rangeand the Great Himalayas.
The Jhelum river rises in the Pir PanjalRange in Jammu and Kashmir, India,and flows north-
westward through thevalley of Kashmir, before entering thePakistani-administered sector.
Thehighest point is the Nanga Parbat, at thenorth-western end of the region.Dalhousie, in
Himachal Pradesh in thefoothills of the range, is a noted hillstation (mountain resort).

8.Answer: (a)
Explanation:
The Assam Himalayas, the easternsection of the Great Himalayas,extending eastward
across Sikkimstate (India) and Bhutan, into northernAssam and Arunachal Pradesh
states (India), and along the border with the Tibet Autonomous Region (China).
The mountains run eastward for 450 miles (720 km) from the Upper Tista river in the west, to
the great southward bend of the Brahmaputra river (there called the Tsangpo river) in the
east. Important peaks include Kula, Chomo and Kangto; the highest is Namjagbarwa in Tibet.
The Subansiri, Manas, Sankosh, Raidak and Jaldhaka rivers rise in the mountains and
flow southward to join the Brahmaputra. Main settlements in the region include Gangtok
and Kalimpang in India; and Punakha and Paro in Bhutan. Important mountain passes
include Natu, Jelep and Tang.
The Barak river rises from the Manipur hills, south of Mao in Senapati district of Manipur. It
flows then along the Nagaland-Manipur border through hilly terrains and enters Assam.

9.Answer: (a)
Explanation:
The Meghalaya Plateau is a highlandregion in eastern Meghalaya. It is arolling tableland
and the highest portionof the hill mass that comprises most ofMeghalaya. The Plateau’s
western,northern and southern escarpments arecalled the Garo, Khasi and Jaintia
hills,respectively.
It is an outlier of the plateau of thePeninsular India and is composedprimarily of ancient
rocks.
In fact, it is an extension of the mainPeninsular Plateau. It is believed thatdue to the force
exerted by the north-eastward movement of the Indian Plate atthe time of the Himalayan
origin, a hugefault was created between the Rajmahalhills and the Meghalaya Plateau.
Later,this depression got filled up by thedeposition activity of numerous rivers. An extension
of this is also seen in the Karbi Anglong hills of Assam. Similar to the Chotanagpur Plateau,
the Meghalaya Plateau is also rich in mineral resources, like coal, iron ore, sillimanite,
limestone and uranium.
This area receives maximum rainfall from the south-west monsoon. As a result, the
Meghalaya plateau has a highly eroded surface. Cherrapunji displays a bare rocky surface,
devoid of any permanent vegetation cover.

10. Answer: (b)


Explanation:
The Kuchchh Peninsula was an island surrounded by seas and lagoons. These seas and
lagoons were later filled by sediment brought by the Indus river, which used to flow through
this area. Thus, the island became a part of the mainland and comparatively a broad plain
was formed. Some scholars do not consider it as a part of the west coastal plains and treat it
as a separate identity.
There are other geographers who consider Kuchchh and Kathiawar as part of the Peninsular
Plateau, because Kathiawar is made of the Deccan lava and there are tertiary rocks in the
Kuchchh area. The true west coastal plain, according to them, lies between Surat and
Kanniyakumari, for a distance of 1,600 km. But the ground reality is that it is, more or less,
a plain area and lies near the west coast of India. Hence, it should be treated as an integral
part of the West Coastal Plains.
Pachham, Khadir and Bela are found in this part.
Kuchchh falls in the seismically active Zone-V of the Indian sub- continent, outside the
Himalayan seismic belt.
The Luni river originates in the Pushkar valley of the Aravalli range, near Ajmer.

11. Answer: (c)


Explanation:
Konkan is the coastal plain of western India, lying between the Arabian Sea (west) and the
Western Ghats (east).
The plain stretches from the Daman Ganga river, north of Mumbai (Bombay) to the Terekhol
river between Maharashtra and Goa states, and Daman and Diu Union Territory in the
south.
The Konkan includes the regions of Thane, Greater Mumbai, Raigarh and Ratnagiri.
The region is traversed by seasonal rivers that drain the heavy monsoonal rainfall from the
crest of the Sahyadri hills.
Rivers like Ulhas, Vaitarna, Zuari and Mandovi drain these plains.
The generally uneven terrain is composed of eroded remnant ranges of the Ghats that form
low lateritic plateaus in the west and terminate in a coastline of alternating bays and
headlands.
Only about one-third of the land is cultivable, and the population lives mainly in the
relatively fertile river valleys near the coast and in the newly developed industrial belts
around Mumbai, Thane, Khopali and Panvel.
The barren hills are occupied by the pastoral Bhil, Kathkari and Kokana people. The main
crops are rice, pulses (legumes), vegetables, fruits and coconuts; fishing and salt
manufacture are also important.

12. Answer: (a)

1
3.
14. Answer: (a)
Explanation:
Bhabar is a narrow belt ranging between8-10 km parallel to the Shiwalik foothillsat the
break-up of the slope. As a result of this, the streams and rivers coming from the mountains
deposit heavy materials of rocks and boulders, and at times, disappear in this
zone. South of the Bhabar is the Tarai belt, with an approximate width of 10-20 km, where
most of the streams and rivers re-emerge without having any properly demarcated channel,
thereby, creating marshy and swampy conditions, known as the Tarai.
This has a luxurious growth of natural vegetation and houses a varied wildlife. The south of
Tarai is a belt consisting of old and new alluvial deposits, known as the Bhangar and the
Khadar, respectively. These plains have characteristic features of mature stage of fluvial
erosional and depositional landforms, such as sand bars, meanders, oxbow lakes and
braided channels.
The Brahmaputra plains are known for their riverine islands and sand bars. Most of these
areas are subjected to periodic floods and shifting river courses, forming braided streams.

15.Answer: (a)
Explanation:
The Peninsular Plateau is an irregulartriangle, known as the PeninsularPlateau. Delhi Ridge
in the north-west(extension of the Aravallis), the Rajmahalhills in the east, Gir Range in the
westand the Cardamom hills in the south,constitute the outer extent of thePeninsular
Plateau. However, anextension of this is also seen in the north- in the form of Shillong and
the Karbi-Anglong Plateau.
The Peninsular India is made up of a series of Patland Plateaus, such as the Hazaribagh
Plateau, the Palamu Plateau, the Ranchi Plateau, the Malwa Plateau, the Coimbatore Plateau
and the Karnataka Plateau, etc.
The general elevation of the Plateau is from the west to the east, which is also proved
by the pattern of the flow of the rivers.
Some of the important physiographic features of this region are tors, block mountains, rift
valleys, spurs, bare rocky structures, series of hummocky hills and wall-like quartzite dykes,
offering natural sites for water storage. The western and north-western parts of the plateau
have an emphatic presence of black soil.
The north-western part of the Plateau has a complex relief of ravines and gorges. The
ravines of Chambal, Bhind and Morena (The Central Highlands) are some of the well-
known examples.

16. Answer: (b)


Explanation:
The general elevation of the Central Highlands ranges between 700-1,000 m above the
mean sea level and they slope towards the north and the north-eastern directions.
Most of the tributaries of river Yamuna have their origin in the Vindhyan and the Kaimur
ranges (The Central Highlands). Banas is the only significant tributary of river Chambal, that
originates from the Aravallis in the west. An eastern extension of the Central Highlands is
formed by the Rajmahal hills, to the south of which lies a large reserve of mineral resources
in the Chotanagpur Plateau.
17. Answer: (c)
Explanation:
To the north-west of the Aravallis hills lies the Great Indian Desert. It is a land of undulating
topography, dotted with longitudinal dunes and barchans. This region receives low rainfall
below 150 mm per year; hence, it has arid climate with low vegetation cover.
Though the underlying rock structure of the desert is an extension of the Peninsular Plateau,
yet, due to extreme arid conditions, its surface features have been carved by physical
weathering and wind actions.
Some of the well pronounced desert land features present here are mushroom rocks, shifting
dunes and oasis (mostly in its southern part). Most of the rivers in this region are ephemeral.
The Luni river, flowing in the southern part of the desert, is of some significance.
The lakes and the playas have brackish water, which is the main source of obtaining salt.

18. Answer: C

19. Answer: c
20. Answer: (a)
Explanation:
The drainage pattern, resembling the branches of a tree, is known as ‘dendritic’,
theexamples of which are the rivers of the northern plains.
When the rivers originate from a hill and flow in all directions, the drainage pattern
isknown as ‘radial’. The rivers originating from the Amarkantak Range present
goodexamples of it.
When the primary tributaries of the river flow parallel to each other and the
secondarytributaries join them at right angles, the pattern is known as ‘trellis’.
When the rivers discharge their waters from all directions in a lake or depression,
thepattern is known as ‘centripetal’.

21. Answer: (d)


Explanation:
22. Answer: (b)
Explanation:
 The Indus, also known as the Sindhu, is the western-most of the Himalayan rivers in India.
 It originates from a glacier near Bokhar Chu in the Tibetan region in the Kailash Mountain
Range.
 In Tibet, it is known as ‘Singi Khamban’, or ‘The Lion’s Mouth’.
 It enters Pakistan near Chilas in the Dardistan region.
 The Indus receives a number of Himalayan tributaries, such as the Shyok, the Gilgit, the
Zaskar, the Hunza, the Nubra, the Shigar, the Gasting and the Dras.
 The other important tributaries joining the right bank of the Indus are the Khurram, the
Tochi, the Gomal, the Viboa and the Sangar.
 The Indus flows in India only through Jammu and Kashmir.
 India constructed the Pakal Dul Hydroelectric Project on the Chenab river.

23. Answer: (b)


Explanation:
 The Jhelum, an important tributary of the Indus, rises from a spring at Verinag, situated at
the foot of the Pir Panjal in the south-eastern part of the valley of Kashmir.
 It flows through Srinagar and the Wular Lake, before entering Pakistan through a deep
narrow gorge.
 It joins the Chenab near Jhang in Pakistan.
 The Kishanganga Hydroelectric Project is a run-of-the-river project on the Kishanganga
tributary of the Jhelum river.
 The Jhelum river is the western-most tributary of the Indus river.

Note: The Chenab is the largest tributary of the Indus river.

24. Answer: (c)


Explanation:
 The Chenab is the largest tributary of the Indus.
 It is formed by two streams, the Chandra and the Bhaga, which join at Tandi near Keylong
in Himachal Pradesh. Hence, it is also known as the Chandrabhaga.
 The river flows for 1,180 km before entering Pakistan.
 In India, the watershed of the Chenab basin covers part of one state, viz., Himachal
Pradesh and one Union Territory, viz., Jammu and Kashmir.
 It was called Asikni in the Rigvedic period.
 The Marusudar is the biggest tributary of the Chenab and meets the Chenab at
Bhandalkot.

25. Answer: (b)


Explanation:
 The Ravi river is another important tributary of the Indus. It rises west of the Rohtang Pass
in the Kullu hills of Himachal Pradesh and flows through the Chamba valley of the state.
 The Beas river is another important tributary of the Indus, originating from the Beas Kund
near the Rohtang Pass at an elevation of 4,000 m above the mean sea level. It enters the
Punjab plains, where it meets the Sutlej near Harike. It passes through the states of Punjab
and Himachal Pradesh.
 The Ravi river is transboundary, but not the Beas river.

26. Answer: (b)


Explanation:
 The Sutlej originates in the ‘Raksas Tal’ near Mansarovar at an altitude of 4,555 m in Tibet,
where it is known as the Langchen Khambab.
 It flows almost parallel to the Indus for about 400 km, before entering India and comes out
of a gorge at Rupar.
 It passes through the Shipki La on the Himalayan Ranges and enters the Punjab plains.
 It is an antecedent river.
 It is a very important tributary, as it feeds the canal system of the Bhakra Nangal Project.
 It meets river Beas near Harike Pattan in Punjab.
27.Answer: (a)
Explanation:
The Ganga river rises in the Gangotriglacier near Gaumukh (3,900 m) inthe Uttarkashi
district ofUttarakhand. Here, it is known as theBhagirathi.
At Dev Prayag, the Bhagirathimeets the Alaknanda. Hereafter, itis known as the
Ganga. TheAlaknanda has its source in theSatopanth glacier above Badrinath.The
Alaknanda consists of the Dhauliand the Vishnu Ganga, which meet atJoshimath, or Vishnu
Prayag.
The other tributaries of theAlaknanda, such as the Pindar, join itat Karna Prayag, while the
Mandakinior the Kali Ganga meets it at RudraPrayag.
The Son is its major right banktributary.
The important left bank tributariesare the Ramganga, the Gomati, theGhaghara, the
Gandak, the Kosi andthe Mahananda.
The river finally discharges itself intothe Bay of Bengal near the SagarIsland.
The Ganga basin is the largest riverbasin in India in terms of catchmentarea. It covers 10
states, viz.,Uttarakhand, Uttar Pradesh, MadhyaPradesh, Rajasthan, Haryana, Himachal
Pradesh, Chhattisgarh, Jharkhand, Bihar and West Bengal, and one Union Territory of Delhi.

28.Answer: (b)
Explanation:
The Yamuna, the western-most andthe longest tributary of the Ganga,has its source
in the Yamunotriglacier on the western slopes of theBanderpunch Range (6,316 m).
It joins the Ganga at Prayag(Allahabad). It is joined by the
Chambal, the Sind, the Betwa and the Ken on its right bank, which originate from the
Peninsular Plateau, while the Hindan, the Rind, the Sengar, the Varuna, etc., join it on its
left bank.

29.Answer: (b)
Explanation:
The Gandak river comprises twostreams, namely the Kaligandak andthe Trishulganga.
It rises in the Nepal Himalayas,between the Dhaulagiri and MountEverest, and drains the
central part ofNepal.
It enters the Ganga plain inChamparan district of Bihar and joinsthe Ganga at Sonpur near
Patna.
The Ghaghara originates in theglaciers of Mapchachungo. Aftercollecting the waters of its
tributaries–the Tila, the Seti and the Beri, itcomes out of the mountain, cutting adeep gorge
at Shishapani.
The Sarda river (Kali or Kali Ganga)joins it in the plain before it finallymeets the Ganga at
Chhapra.
Both the Gandak and the Ghagharaare the left bank tributaries of theGanga river.

30.Answer: (c)
Explanation:
The Northern Plains, from the north tothe south, can be divided into threemajor zones: the
Bhabar, the Taraiand the alluvial plains.
South of the Bhabar is the Tarai belt,with an approximate width of 10-20km, where most of
the streams andrivers re-emerge without having anyproperly demarcated channel,thereby,
creating marshy and swampyconditions, known as the Tarai. Thishas a luxurious growth of
naturalvegetation and houses a variedwildlife.
 The south of Tarai is a belt consisting of old and new alluvial deposits, known as the
Bhangar and the Khadar, respectively.
31. Answer: (a)
Explanation:
The Northern Plains are formed by the alluvial deposits brought by the rivers – the
Indus, the Ganga and the Brahmaputra.
The states of Haryana and Delhi form a water divide between the Indus and the Ganga river
systems.
Note: The Peninsular Plateau is one of the oldest and most stable landmass of India.
32. Answer: (c)
Explanation:
 Rising from the height of 150 m above the river plains, upto an elevation of 600-900 m, is
the irregular triangle, known as the Peninsular Plateau.
 Delhi Ridge in the north-west, (extension of the Aravalli), the Rajmahal Hills in the east, the
Gir Range in the west and the Cardamom Hills in the south, constitute the outer extent of
the Peninsular Plateau.
 However, an extension of this is also seen in the north-east, in the form of Shillong and the
Karbi-Anglong Plateau.
 This is one of the oldest and the most stable landmass of India.
 The general elevation of the Plateau is from the west to the east, which is also
proved by the pattern of the flow of the rivers.
 Some of the important physiographic features of this region are tors, block mountains, rift
valleys, spurs, bare rocky structures, series of hummocky hills and wall-like quartzite dykes,
offering natural sites for water
storage. The western and the north-western part of the Plateau has an emphatic presence of
black soil.

33. Answer: (b)


Explanation:
The Deccan Plateau:
 It is bordered by the Western Ghats in the west, the Eastern Ghats in the east and the
Satpura, Maikal Range and the Mahadeo Hills in the north.
 The Western Ghats are locally known by different names, such as Sahyadri in
Maharashtra, the Nilgiri Hills in Karnataka and Tamil Nadu, and the Anaimalai Hills and the
Cardamom Hills in Kerala.
 The Western Ghats are comparatively higher in elevation and more continuous than the
Eastern Ghats.
 ‘Anaimudi’ (2,695 m), the highest peak of the Peninsular Plateau, is located on the
Anaimalai Hills of the Western Ghats, followed by Dodabetta (2,637 m) on the Nilgiri
Hills.
 Most of the Peninsular rivers have their origin in the Western Ghats.
 The Eastern Ghats, comprising discontinuous and low hills, are highly eroded by the
rivers, such as the Mahanadi, the Godavari, the Krishna, the Kaveri, etc.
 The Eastern and the Western Ghats meet each other at the Nilgiri Hills.

34. Answer: (c)


Explanation:
From east to west direction:
Namcha Barwa – Kanchenjunga – Kamet – Nanga Parbat

35. Answer: (a)


Explanation:
From west to east direction:
Satmala – Kaimur – Garhjat – Rajmahal

36.Answer: (b)
Explanation:

37. Answer: (c)


Explanation:
The Tropic of Cancer passes throughthe following states:
1.Gujarat
2.Rajasthan
3.Madhya Pradesh
4.Chhattisgarh
5.Jharkhand
6.West Bengal
7.Tripura
8.Mizoram
Note: The Tropic of Cancer does not pass through the state of Odisha.

38.Answer: (b)
Explanation:
The Kosi is an antecedent river with itssource to the north of Mount Everest inTibet, where
its main stream Arun rises.After crossing the Central Himalayas inNepal, it is joined by the
Son Kosi fromthe west and the Tamur Kosi from theeast. It forms the Sapt Kosi after
unitingwith river Arun.
The Ramganga is comparatively asmall river rising in the Garhwal Hillsnear Gairsain. It
changes its course tothe south-west direction after crossingthe Shiwalik and enters the
plains ofUttar Pradesh near Najibabad. Finally, itjoins the Ganga near Kannauj.
The Damodar occupies the easternmargins of the Chotanagpur Plateau,where it flows
through a rift valley and finally joins the Hugli. The Barakar is its main tributary. Once
known as the ‘Sorrow of Bengal’, the Damodar has been now tamed by the Damodar Valley
Corporation, a multi-purpose project.
The Damodar and the Barakar trifurcate the Chotanagpur Plateau.

39.Answer: (c)
Explanation:
The Sarda, or Saryu river rises in theMilam Glacier in the Nepal Himalayas,where it is known
as the Goriganga.Along the Indo-Nepal border, it is calledKali or Chauk, where it joins
theGhaghara.
The Mahananda is another importanttributary of the Ganga, rising in theDarjeeling Hills. It
joins the Ganga as itslast left bank tributary in West Bengal.
The Son is a large south bank tributary ofthe Ganga, originating in the AmarkantakPlateau.
After forming a series ofwaterfalls at the edge of the Plateau, itreaches Arrah, west of Patna,
to join theGanga.

40.Answer: (c)
Explanation:
The Brahmaputra river, one of the largestrivers of the world, has its origin in
theChemayungdung Glacier of the KailashRange near the Mansarovar Lake. Fromhere, it
traverses eastward longitudinallyfor a distance of nearly 1,200 km in a dryand flat region of
southern Tibet, where itis known as the Tsangpo, which means‘The Purifier.’ The Rango
Tsangpo is themajor right bank tributary of this river inTibet. It emerges as a turbulent
anddynamic river after carving out a deepgorge in the Central Himalayas nearNamcha Barwa
(7,755 m).
The river emerges from the foothills underthe name of Siang or Dihang. It entersIndia west of
Sadiya town in Arunachal Pradesh. Flowing south-west, it receives its main left bank
tributaries, viz., the Dibang or Sikang and the Lohit; thereafter, it is known as the
Brahmaputra.
The Brahmaputra receives numerous tributaries in its 750 km long journey through the
Assam valley. Its major left bank tributaries are the Burhi Dihing and the Dhansari (South),
whereas the important right bank tributaries are the Subansiri, the Kameng, the Manas and
the Sankosh. The Subansiri, which has its origin in Tibet, is an antecedent river. The
Brahmaputra enters Bangladesh near Dhubri and flows southward.

41. Answer: (b)


Explanation:
Three major geological events in the distant past have shaped the present drainage systems
of the Peninsular India:
(i) Subsidence of the western flank of the Peninsula, leading to its submergence below the
sea during the early tertiary period. Generally, it has disturbed the symmetrical plan of the
river on either side of the original watershed.
(ii) Upheaval of the Himalayas, when the northern flank of the Peninsular block was
subjected to subsidence and the consequent trough faulting. The Narmada and the Tapi flow
in trough faults and fill the original cracks with their detritus materials. Hence, there is a
lack of alluvial and deltaic deposits in these rivers.
(iii) Slight tilting of the Peninsular block from the north-west to the south-eastern
direction gave orientation to the entire drainage system towards the Bay of Bengal during
the same period.

42. Answer: (a)


Explanation:
Most of the major Peninsular rivers, except the Narmada and the Tapi, flow from west to east.
The Chambal, the Sind, the Betwa, the Ken and the Son, originating in the northern part of
the Peninsula, belong to the Ganga river system. The other major river systems of the
Peninsular drainage are – the Mahanadi, the Godavari, the Krishna and the Kaveri.
Peninsular rivers are characterised by fixed course, absence of meanders and non-perennial
flow of water. The Narmada and the Tapi, which flow through the rift valley are, however,
exceptions.
The Peninsular drainage system is older than the Himalayan one. This is evident from the
broad, largely-graded shallow valleys, and the maturity of the rivers. Whereas, the rivers in
the Himalayan drainage pass through the giant gorges carved out by the erosional activity
carried on simultaneously with the uplift of the Himalayas. Besides deep gorges, these
rivers also form V-shaped valleys, rapids and waterfalls in their mountainous course.
43. Answer: (b)
Explanation:
The Godavari is the largest Peninsular river system. It is also called the Dakshin Ganga.
It rises in the Nasik district of Maharashtra and discharges its water into the Bay of Bengal.
Its tributaries run through the states of Maharashtra, Madhya Pradesh, Chhattisgarh,
Odisha and Andhra Pradesh. It is 1,465 km long with a catchment area spreading over 3.13
lakh sq. km. 49 per cent of this lies in Maharashtra, 20 per cent in Madhya Pradesh and
Chhattisgarh, and the rest in Andhra Pradesh. The Penganga, the Indravati, the Pranhita
and the Manjra are its principal tributaries. The Godavari is subjected to heavy floods in
its lower reaches to the south of Polavaram, where it forms a picturesque gorge. It is
navigable only in the deltaic stretch. The river after Rajamundri splits into several branches,
forming a large delta.
The Bhavani is an important tributary of the Kaveri.

44. Answer: (a)


Explanation:
The Krishna is the second largest east-flowing Peninsular river, which rises near
Mahabaleshwar in Sahyadri. Its total length is 1,401 km. The Koyna, the Tungbhadra and
the Bhima are its major tributaries. Of the total catchment area of the Krishna, 27 per cent
lies in Maharashtra, 44 per cent in Karnataka and 29 per cent in Andhra Pradesh and
Telangana.
The Kaveri rises in the Brahmagiri Hills (1,341m) of Kogadu district in Karnataka. Its
length is 800 km and it drains an area of 81,155 sq. km. Since the upper catchment area
receives rainfall during the south-west monsoon season (summer) and the lower part during
the north-east monsoon season (winter), the river carries water throughout the year with
comparatively less fluctuation than the other Peninsular rivers. About 3 per cent of the
Kaveri basin falls in Kerala, 41 per cent in Karnataka and 56 per cent in Tamil Nadu. Its
important tributaries are the Kabini, the Bhavani and the Amravati.

45. Answer: (c)


Explanation:
The Narmada originates on the western flank of the Amarkantak Plateau. Flowing in a rift
valley between the Satpura Range in the south and the Vindhyan Range in the north, it forms
a picturesque gorge in marble rocks and Dhuandhar waterfall near Jabalpur. The Sardar
Sarovar Project has been constructed on this river.
It is one of only two major rivers in the Peninsular India that runs from east to west (longest
west-flowing river), along with the Tapti river. It is one of the rivers in India that flows in a rift
valley, bordered by the Satpura and the Vindhyan Ranges. As a rift valley river, the Narmada
does not form a delta. The rift valley rivers form estuaries.
The Tapi is the other important westward flowing river. It originates from Multai in the Betul
district of Madhya Pradesh. It is 724 km long and drains an area of 65,145 sq. km. Nearly 79
per cent of its basin lies in Maharashtra, 15 per cent in Madhya Pradesh and the remaining 6
per cent in Gujarat.

46. Answer: (b)


Explanation:
The Mahanadi rises near Sihawa in Raipur district of Chhattisgarh, and runs through Odisha
to discharge its water into the Bay of Bengal. It is 851 km long and its catchment area
spreads over 1.42 lakh sq. km. Fifty-three per cent of the drainage basin of this river lies
in Madhya Pradesh and Chhattisgarh, while 47 per cent lies in Odisha.
Luni is the largest river system of Rajasthan, west of the Aravalli. It originates near Pushkar
in two branches, viz., the Saraswati and the Sabarmati, which join with each other at
Govindgarh. From here, the river comes out of the Aravalli and is known as Luni. It flows
towards the west till Telwara and then takes a south-west direction to join the Rann of
Kuchchh. The entire river system is ephemeral.

47.Answer: (d)
Explanation:
The Kanwar lake in Bihar, Asia’s largestfreshwater oxbow lake.
It is a residual oxbow lake, formed due tomeandering of Gandak river, a tributaryof Ganga, in
the geological past.
It was declared a notified area under theWildlife (Protection) Act of 1972.

48.Answer: (c)
Explanation:
Biligirirangan Hills is located at thenorth-west of the Western Ghats andwesternmost edge
of the Eastern Ghats, itsupports a diverse flora and fauna in viewof the various habitats
present.
Seshachalam Hills- It is a hill ranges ofthe Eastern Ghats, southern AndhraPradesh state,
southeastern India. Theranges contain sandstone and shaleinterbedded with limestone and
arehighly dissected, with many longitudinalvalleys. Tributaries of the Penneru Riverdrain the
region.

49.Answer: (c)
Explanation:
The plateau is an extension of the DeccanTraps, formed at the end ofthe Cretaceous period.
In this region themain classes of soil are black, brownand bhatori (stony) soil.
The volcanic, clay-like soil of the region owes its black colour to the high iron content of the
basalt from which it formed. The soil requires less irrigation because of its high capacity for
moisture retention. The other two soil types are lighter and have a higher proportion of sand.
The average elevation of the plateau is 500 m. Some of the peaks Sigar, Janapav and
Ghajari.

50.Answer: (a)
Explanation:
Glacier Location
(Karakoram
range)
1. Siachen Nubra
Valley
2. Biafo Braldoh
Valley
3. Yarkand Shyok
Rimo Valley

51. Answer: (a)


Explanation:
The Mechanism of Weather in the Winter Season: In the winter months, the weather
conditions over India are generally influenced by the distribution of pressure in Central and
Western Asia. A high-pressure centre in the region lying to the north of the Himalayas
develops during winter. This centre of high pressure gives rise to the flow of air at the low
level from the north towards the Indian sub-continent, south of the mountain range. The
surface winds blowing out of the high-pressure centre over Central Asia reach India in the
form of a dry continental air mass in a north-westerly direction.
The western cyclonic disturbances, which enter the Indian sub-continent from the west
and the north-west during the winter months, originate over the Mediterranean Sea
and are brought into India by the westerly jet stream. An increase in the prevailing night
temperature generally indicates an advance in the arrival of these cyclonic disturbances.

52. Answer: (c)


Explanation:
As the summer sets in and the Sun shifts northwards, the wind circulation over the Indian
sub-continent undergoes a complete reversal at both, the lower, as well as the upper levels.
By the middle of July, the low pressure belt nearer the surface [termed as the Inter Tropical
Convergence Zone (ITCZ)] shifts northwards, roughly parallel to the Himalayas, between
20° N and 25° N. By this time, the westerly jet stream withdraws fromthe Indian region. The
ITCZ, being a zone of low pressure, attracts inflow of winds from different directions.
The maritime tropical airmass (mT) from the southern hemisphere, after crossing the
Equator, rushes to the low pressure area in the general south-westerly direction. It is this
moist air current which is popularly known as the ‘south-west monsoon’.
An easterly jet stream flows over the southern part of the Peninsula in June, and has a
maximum speed of 90 km per hour.

53. Answer: (a)


Explanation:
There seem to be two rain-bearing systems in India. The first one originates in the Bay of
Bengal, causing rainfall over the plains of north India. The second is the Arabian Sea current
of the south-west monsoon, which brings rain to the west coast of India. Much of the
rainfall along the Western Ghats is orographic, as the moist air is obstructed and forced
to rise along the Ghats.
The onset of monsoon is first experienced by Andaman & Nicobar islands in India. As shown
in the map below.

54. Answer: (b)


Explanation:
The Peninsular region of India does not have any well-defined cold weather season.
There is hardly any seasonal change in the distribution pattern of the temperature in the
coastal areas, because of moderating influence of the sea and the proximity to the Equator.
During the winters, the weather in India is pleasant. Winter monsoons do not cause rainfall,
as they move from the land to the sea. It is because firstly, they have little humidity; and
secondly, due to anti-cyclonic circulation on land, the possibility of rainfall from them
reduces.
During October and November, the north-east monsoon, while crossing over the Bay of
Bengal, picks up moisture and causes torrential rainfall over the Tamil Nadu coast, southern
Andhra Pradesh, south-east Karnataka and south-east Kerala.

55. Answer: (b)


Explanation:
The retreating south-west monsoon season is marked by clear skies and rise in
temperature. The land is still moist. Owing to the conditions of high temperature and
humidity, the weather becomes rather oppressive. This is commonly known as the ‘October
Heat’. In the second half of October, the mercury begins to fall rapidly, particularly in
northern India.
The weather in the retreating monsoon is dry in north India, but it is associated with
rain in the eastern part of the Peninsula. Here, October and November are the rainiest
months of the year. The widespread rain in this season is associated with the passage of
cyclonic depressions, which originate over the Andaman Sea and manage to cross the eastern
coast of the southern Peninsula.

56. Answer: (d)


Explanation:
 Latitude: The northern part of India lies in the sub-tropical and temperate zone, and the
part lying south of the Tropic of Cancer falls in the tropical zone, thus bringing climatic
diversity in India.
 The Himalayan Mountains: These provide an invincible shield to protect the sub-
continent from the cold northern winds. The Himalayas also trap the monsoon winds, forcing
them to shed their moisture within the sub-continent.
 Distribution of Land and Water: As compared to the landmass, water heats up or cools
down slowly. This differential heating of land and sea creates different air pressure zones in
different seasons in and around the Indian sub-continent.
 Distance from the Sea: The areas in the interior of India are far away from the moderating
influence of the sea. Such areas have extremes of climate.
 Altitude: Temperature decreases with height. Due to thin air, places in the mountains are
cooler than the places on the plains.
 Relief: The physiography or relief of India also affects the temperature, air pressure,
direction and speed of wind, and the amount and distribution of rainfall. The windward sides
of the Western Ghats and Assam receive high rainfall.

57. Answer: (b)


Explanation:
El Niño is the anomalous appearance, every few years, of unusually warm ocean conditions
along the tropical west coast of South America. When this warming occurs, the usual
upwelling of cold, nutrient rich deep ocean water is significantly reduced. This event is
associated with adverse effects on fishing, agriculture and local weather from Ecuador to
Chile, and with far-field climatic anomalies in the equatorial Pacific, and occasionally in Asia
and North America as well.
The normal low pressure system is replaced by a weak high in the western Pacific. This
change in the pressure pattern causes the trade winds to weaken.
India generally experiences drought during the El Niño years. However, other factors,
like the Indian Ocean Dipole (IOD) also play their role. There have been El Niño years
during which India has not experienced droughts because of a positive IOD.
58. Answer: (c)
Explanation:
The Madden-Julian Oscillation (MJO) is an oceanic-atmospheric phenomenon which affects
weather activities across the globe. It brings major fluctuation in the tropical weather on
weekly to monthly time-scales.
The MJO can be defined as an eastward moving 'pulse' of clouds, rainfall, winds and
pressure near the Equator, that typically recurs every 30 to 60 days. It is a traversing
phenomenon and is most prominent over the Indian and the Pacific Oceans.
When it is over the Indian Ocean during the monsoon season, it brings good rainfall over the
Indian sub-continent. On the other hand, when it witnesses a longer cycle and stays over the
Pacific Ocean, the MJO brings bad news for the Indian monsoon.

59. Answer: (b)


Explanation:
A cloudburst is defined as an intense downpour that lasts only for a brief time and is
occasionally accompanied by hail and thunder. It is defined by the India Meteorological
Department (IMD) as unexpected precipitation that exceeds 100 mm (or 10 cm) per hour
across a region of land, that is between 20 and 30 square kilometres.
It is a sudden, very heavy rainfall, usually local in nature and of brief duration. Most so-
called cloudbursts occur in connection with thunderstorms. In these storms there are
violent uprushes of air, which, at times, prevent the condensing rain-drops from falling
to the ground. A large amount of water may thus accumulate at high levels, and if the
upward currents are weakened, the whole of this water falls at one time.
Cloudbursts are especially common in the mountainous areas. This is probably because the
warm air currents of a thunderstorm tend to follow the upward slope of a mountain. The
effects of heavy rain are especially striking on the mountain slopes, because the falling water
is concentrated in the valleys and gulleys.

60. Answer: (d)


Explanation:
A landslide is defined as the movement of a mass of rock, debris, or earth down a slope.
Landslides are a type of "mass wasting," which denote any down-slope movement of soil and
rock under the direct influence of gravity.
The causes include the factors that increase the effects of down-slope forces and factors that
contribute to low or reduced strength. Landslides can be initiated in the slopes already on the
verge of movement by rainfall, snowmelt, changes in water level, stream erosion, changes in
ground water, earthquakes, volcanic activity, disturbance by human activities (Mining,
Nuclear tests, Dam building), or any combination of these factors . Earthquake shaking and
other
factors can also induce landslides underwater. These landslides are called submarine
landslides. Submarine landslides sometimes cause Tsunamis, that damage the coastal areas.

61. Answer: (a)


Explanation:
The Ring of Fire, also referred to as the Circum-Pacific Belt, is a path along the Pacific Ocean
characterized by active volcanoes, submarine landslides and frequent earthquakes. Its length
is approximately 40,000 kilometres (24,900 miles). It traces boundaries between several
tectonic plates—including the Pacific, Juan de Fuca, Cocos, Indian-Australian, Nazca, North
American and Philippine Plates.
Seventy-five percent of Earth’s volcanoes—more than 450 volcanoes—are located along the
Ring of Fire. Ninety percent of Earth’s earthquakes occur along its path, including the
planet’s most violent and dramatic seismic events.
The Indus-Tsangpo Suture Zone is formed due to continent-continent convergence and
hence, it lacks active volcanoes.
62. Answer: (c)
Explanaiton:
Tsunami, also called seismic sea wave, catastrophic ocean wave, is usually caused by a
submarine earthquake, an underwater or coastal landslide, or a volcanic eruption. After
an earthquake or other generating impulse occurs, a train of simple, progressive oscillatory
waves is propagated great distances over the ocean surface in ever-widening circles, much
like the waves produced by a pebble falling into a shallow pool. In deep water a Tsunami can
travel as fast as 800 km (500 miles) per hour.

63. Answer: (c)


Explanation:
The parallel orientation of the Tamil Nadu coast does not allow the south-west branch of
monsoon carrying moisture to be obstructed. Winds move along the coast without shedding
their moisture.
The south-west branch, which strikes the Western Ghats from the Arabian Sea, gets dry once
it crosses the Western Ghats. As Tamil Nadu lies into the rain shadow area of the Western
Ghats, it experiences these dry winds again devoid of any moisture.

64. Answer: (b)


Explanation:
 Mango Shower: Towards the end of summer, there are pre-monsoon showers which are a
common phenomenon in Kerala and the coastal areas of Karnataka. Locally, they are known
as ‘mango showers’, since they help in the early ripening of mangoes.
 Blossom Shower: With this shower, coffee flowers blossom in Kerala and the nearby areas.
 Nor Westers: These are the dreaded evening thunderstorms in Bengal and Assam.
Their notorious nature can be understood from the local nomenclature of ‘Kalbaisakhi’, a
calamity of the month of Baisakh. These showers are useful for tea, jute and rice cultivation.
In Assam, these storms are known as “Bardoli Chheerha”.
 Loo: Hot, dry and oppressing winds blowing in the northern plains from Punjab to
Bihar, with higher intensity between Delhi and Patna.

65. Answer: (b)


Explanation:
 The tropical evergreen forests are found in the western slope of the Western Ghats,
hills of the north-
eastern region and the Andaman & Nicobar Islands.
 They are found in warm and humid areas with an annual precipitation of over 200 cm and
mean annual temperature above 22 degrees Celsius. The Tropical Evergreen forests are well
stratified, with layers closer to the ground and are covered with shrubs and creepers, with
short structured trees followed by tall variety of trees.
 In these forests, the trees reach great heights upto 60 m or above. There is no definite time
for the trees to shed their leaves, flowering and fruition. As such these forests appear green
all year round. Species found in these forests include rosewood, mahogany, aini, ebony, etc.

66. Answer: (c)


Explanation:
The semi-evergreen forests are found in the less rainy parts of the western slope of the
Western Ghats, hills of the north-eastern region and the Andaman & Nicobar Islands.
Such forests have a mixture of evergreen and moist deciduous trees. The under-growing
climbers provide an evergreen character to these forests. The main species are white cedar,
hollock and kail.
67. Answer: (c)
Explanation:
 In the tropical thorn forests, the plants remain leafless for most part of the year and give
an expression of scrub vegetation. Important species found are babool, ber, and wild date
palm, khair, neem, khejri, palas, etc. Tussocky grass grows upto a height of 2 m as the under
growth.
 The tropical thorn forests occur in the areas which receive rainfall less than 50 cm. These
consist of a variety of
grasses and shrubs. It includes the semi-arid areas of south-west Punjab, Haryana,
Rajasthan, Gujarat, Madhya Pradesh and Uttar Pradesh.

68. Answer: (a)


Explanation:
 In the mountainous areas, the decrease in temperature with increasing altitude leads to a
corresponding change in natural vegetation. Mountain forests can be classified into two
types, the northern mountain forests and the southern mountain forests. The Himalayan
ranges show a succession of vegetation from the tropical to the tundra, which changes in
with altitude. Deciduous forests are found in the foothills of the Himalayas.
 The southern slopes of the Himalayas carry a thicker vegetation cover, because of
relatively higher precipitation, than the drier north-facing slopes. At higher altitudes,
mosses and lichens form part of the tundra vegetation.
 The southern mountain forests include the forests found in three distinct areas of
the Peninsular India, viz., the Western Ghats, the Vindhyas and the Nilgiris. As they are
closer to the tropics, and only 1,500 m above the sea level, vegetation is temperate in the
higher regions, and sub-tropical on the lower regions of the Western Ghats, especially in
Kerala, Tamil Nadu and Karnataka. The temperate forests are called Sholas in the Nilgiris,
Anaimalai and Palani hills.

69. Answer: (b)


Explanation:
 The moist deciduous forests are more pronounced in the regions which record rainfall
between 100-200 cm.
These forests are found in the north-eastern states along the foothills of the Himalayas and,
the eastern slopes of the Western Ghats and Odisha.
 Tropical deciduous forests are the most widespread forests in India. They are also called
the monsoon forests. They spread over regions which receive rainfall between 70-200 cm. On
the basis of the availability of water, these forests are further divided into moist and dry
deciduous.
 Teak, sal, shisham, hurra, mahua, amla, semul, kusum and sandalwood etc., are the
main species of the moist deciduous forests.

70. Answer: (c)


Explanation:
 Dry deciduous forests cover vast areas of the country, where rainfall ranges between 70 -
100 cm. On the wetter
margins, it has a transition to the moist deciduous, while on the drier margins to the thorn
forests. These forests are found in the rainier areas of the Peninsula, and the plains of Uttar
Pradesh and Bihar. In the higher rainfall regions of the Peninsular Plateau and the northern
Indian plain, these forests have a parkland landscape with open stretches, in which teak and
other trees interspersed with patches of grass are common.
 As the dry season begins, the trees shed their leaves completely and the forest appears like
a vast grassland with naked trees all around. Tendu, palas, amaltas, bel, khair and
axlewood, etc., are the common trees of these forests. In the western and southern parts of
Rajasthan, vegetation cover is very scanty due to low rainfall and over-grazing.
71. Answer: (a)
Explanation:

7. Great Nicobar (885) 06.01.1989 Southern most


island of Andaman
and Nicobar
Islands.
8. Similipal (4374) 21.06.1994 Part of Mayurbhanj
District in Odisha.
9. Dibru-Saikhowa 28.07.1997 Part of Dibrugarh
(765) and Tinsukia
Districts in Assam
10 Dehang Debang 02.09.1998 Part of Upper Siang,
(5111.5) West Siang and
Dibang Valley
Districts in
Arunachal Pradesh.
11. Pachmarhi 03.03.1999 Part of Betul,
(4981.72) Hoshangabad and
Chhindwara
Districts in Madhya
Pradesh.
12. Khangchendzonga 07.02.2000 Part of North and
(2619.92) West Districts in
Sikkim
13. Agasthyamalai 12. 11.2001 Part of Thirunelveli
(3500.36) and Kanyakumari
Districts in Tamil
Nadu and
Thiruvananthapura
m, Kollam and
Pathanmthitta
districts in Kerala.
14. Achanakmar 30.03.2005 Part of Anuppur
Ainarkaritak and Dindori
(3835.51) Districts of Madhya
Pradesh and
Bilaspur district of
Chhattisgarh
15. Kachchh (12,454) 29.01.2008 Part of Kachchh,
Rajkot,
Surendranagar and
Patan Districts in
Gujarat.
16. Cold Desert (7770) 28.08.2009 Pin Valley National
Park and
surroundings;
Chandratal and
Sarchu and Kibber
Wildlife sanctuary
in Himachal
Pradesh.
17. Seshachalam 20.09.2010 Seshachalam hill
(4755.997) ranges in Eastern
Ghatsencompassing
part of Chittoor and
Kadapa Districts in
Andhra Pradesh.
18. Panna (2998.98) 25.08.2011 Part of Pann and
Chhattarpur
Districts in Madhya
Pradesh.

72. Answer: (b)


Explanation:
 The mangrove forest and other wetlands of the island arcs of the Andaman and Nicobar
Islands. Mangroves grow along the coasts in the salt marshes, tidal creeks, mud flats and
estuaries.
 They are characterized by halophytic (salt loving) trees, shrubs and other plants growing in
brackish to saline tidal waters.
 Criss-crossed by creeks of stagnant water and tidal flows, these forests give shelter to a
wide variety of birds.
 In India, the mangrove forests spread over 6,740 sq. km, which is 7 per cent
of the world’s mangrove forests. They are highly developed in the Andaman & Nicobar Islands
and the Sundarbans of West Bengal. Other areas of significance are the Mahanadi, the
Godavari and the Krishna deltas. These forests too, are being encroached upon, and hence,
need conservation.
 The Baratang Island mangrove is located in the Andaman and Nicobar Islands.
Note: Underground tissue of any plant requires oxygen for respiration and in the mangrove
environment, oxygen in the soil is very limited or nil. This necessitates the mangrove root
system to take up oxygen from the atmosphere. For this purpose, the mangrove species have
specialized above the ground roots, called the breathing roots or the pneumatophores. In
some species, these roots are pencil sized and peg like, whereas in some other species they
look like a knee. These roots have numerous pores through which oxygen enters into the
underground tissues. In some plants, buttress roots function as breathing roots and also
provide mechanical support to the tree.

73. Answer: (a)


Explanation:
Soil:
● Wheat is grown in a variety of soils of India. Soils with a clay loam or loam texture, good
structure and moderate water holding capacity are ideal for wheat cultivation. Care should be
taken to avoid very porous and excessively drained soils. Soil should be neutral in its
reaction.
● Heavy soil with good drainage is suitable for wheat cultivation under the dry conditions.
These soils absorb and retain rainwater well. Heavy soils with poor structure and poor
drainage are not suitable, as wheat is sensitive to water logging. Wheat can be
successfully grown on lighter soils, provided their water and nutrient holding capacity is
improved.

Climate Requirement:
● Wheat is mainly a rabi (winter) season crop in India.
● Wheat crops have wide adaptability. They can be grown not only in the tropical and
sub-tropical zones, but also in the temperate zone and the cold tracts of the far north,
beyond even the 60 degrees north altitude. Wheat can tolerate severe cold and snow, and
resume growth with the setting in of warm weather in spring. It can be cultivated from sea
level to as high as 3,300 metres.
● The best wheat is produced in the areas favoured with cool, moist weather during the major
portion of the growing period, followed by dry, warm weather to enable the grain to ripen
properly. The optimum temperature range for ideal germination of wheat seed is 20-25
degrees C, though the seeds can germinate in the temperature range 3.5 to 35 degrees C.
● Rains just after sowing hamper germination and encourage seedling blight. The areas with
a warm and damp climate are not suited for wheat growing.
● During the heading and flowering stages, excessively high or low temperatures and drought
are harmful to wheat. Cloudy weather, with high humidity and low temperatures is
conducive for rust attack.
● Wheat plants require about 14-15 degrees C optimum average temperature at the
time of ripening. The temperature conditions at the time of grain filling and
development are very crucial for yield. Temperatures above 25 degrees C during this

period tend to depress the grain weight. When temperatures are high, too much energy
is lost through the process of transpiration by the plants and the reduced residual
energy results in poorer grain formation and lower yields.

74. Answer: (b)


Explanation:
Millets:
● Jowar, bajra and ragi are the important millets grown in India. Though, these are known as
coarse grains, they have very high nutritional value. For example, ragi is very rich in iron,
calcium, other micro nutrients and roughage.
● Jowar is the third most important food crop with respect to area and production. It is a
rain-fed crop, mostly grown in the moist areas which hardly need irrigation. Major jowar
producing states are Maharashtra, Karnataka, Andhra Pradesh and Madhya Pradesh.
● Bajra grows well on sandy soils and shallow black soil. Major bajra producing states are
Rajasthan, Uttar Pradesh, Maharashtra, Gujarat and Haryana.
● Ragi is a crop of dry regions and grows well on red, black, sandy, loamy and shallow
black soils. Major ragi producing states are Karnataka, Tamil Nadu, Himachal Pradesh,
Uttarakhand, Sikkim, Jharkhand and Arunachal Pradesh.

75. Answer: (b)


Explanation:
Maize:
● It is a crop which is used both as food and fodder. It is a kharif crop which requires
temperature between 21°C
and 27°C, and grows well in old alluvial soil.
● In some states, like Bihar, maize is grown in the rabi season also. Use of modern
inputs, such as HYV (High-yielding variety) seeds, fertilisers and irrigation have contributed
to the increasing production of maize.
● Major maize-producing states are Karnataka, Madhya Pradesh, Uttar Pradesh, Bihar,
Andhra Pradesh and Telangana.

Maize can be grown successfully in a variety of soils, ranging from loamy sand to clay loam.
However, soils with good organic matter content, having high water holding capacity with
neutral pH, are considered good for higher productivity. Being a sensitive crop to moisture
stress, particularly excess soil moisture and salinity stresses, it is desirable to avoid low lying
fields having poor drainage and also the fields having higher salinity. Therefore, the fields
having provision of proper drainage should be selected for the cultivation of maize.

76. Answer: (d)


Explanation:
Pulses:
● India is the largest producer, as well as the consumer of pulses in the world. These are the
major source of protein in a vegetarian diet. Major pulses, that are grown in India, are tur
(arhar), urad, moong, masur, peas and gram.
● Pulses need less moisture and survive even in dry conditions. Being leguminous crops, all
these crops, except arhar, help in restoring soil fertility by fixing nitrogen from the air.
● Therefore, these are mostly grown in rotation with other crops. Major pulse producing
states in India are Madhya
Pradesh, Rajasthan, Maharashtra, Uttar Pradesh and Karnataka.
Pulses are grown in all the three seasons. The three crop seasons for the commodity are:
● Kharif: Arhar (Tur); Urd (Blackgram); Moong (Greengram); Lobia (Cowpea); Kulthi
(Horsegram); and Moth.
● Rabi: Gram; Lentil; Pea; Lathyrus; and Rajmah.
● Summer: Greengram; Blackgram; and Cowpea.

77. Answer: (c)


Explanation:
Sugarcane:
● Sugarcane originated in New Guinea, where it has been known for thousands of years.
● It is a tropical, as well as a sub-tropical crop. It grows well in hot and humid climates with
a temperature of 21°C to 27°C and an annual rainfall between 75 cm and 100 cm. Irrigation
is required in the regions of low rainfall.
● It can be grown on a variety of soils and needs manual labour from sowing to harvesting.
● India is the second largest producer of sugarcane, only after Brazil.
● It is the main source of sugar, gur (jaggery), khandsari and molasses. The major
sugarcane-producing states are Uttar Pradesh, Maharashtra, Karnataka, Tamil Nadu,
Andhra Pradesh, Telangana, Bihar, Punjab and Haryana.
● In the southern states, the yield level of sugarcane is high. Its yield is low in northern India.
● The sugarcane productivity and juice quality are profoundly influenced by the weather
conditions prevailing during the various crop-growth sub-periods. Sugar recovery is the
highest when the weather is dry with low
humidity; bright sunshine hours, cooler nights with wide diurnal variations and very little
rainfall during the ripening period. These conditions favour high sugar accumulation. The
climatic conditions, like very high temperature or very low temperature, deteriorate the juice
quality and thus, affecting the sugar quality.

78. Answer: (c)


Explanation:
Oil Seeds:
● Different oil seeds are grown covering approximately 12 percent of the total cropped area of
the country.
● Majority of the oilseeds are cultivated under the rain-fed ecosystem (70%).
● Main oil-seeds produced in India are groundnut, mustard, coconut, sesamum (til),
soyabean, castor seeds, cotton seeds, linseed and sunflower. Most of these are edible and
used as cooking mediums.
● However, some of these are also used as raw material in the production of soaps, cosmetics
and ointments.
● Groundnut is a kharif crop and accounts for about half of the major oilseeds produced in
the country. Gujarat was the largest producer of groundnut, followed by Rajasthan and Tamil
Nadu in 2019–20.
● Linseed and mustard are rabi crops.
● Sesamum is a kharif crop in north India and rabi crop in south India.
● Castor seed is grown both as rabi and kharif crop.

79. Answer: (a)


Explanation:
Tea:
● Tea cultivation is an example of plantation agriculture. It is also an important beverage
crop introduced in
India initially by the British. Today, most of the tea plantations are owned by the Indians.
● The tea plant grows well in the tropical and sub-tropical climates, endowed with deep
and fertile well-drained soil, rich in humus and organic matter.
● Tea bushes require a warm and moist frost-free climate all through the year. Frequent
showers evenly distributed over the year ensure continuous growth of tender leaves.
● Tea is a labour-intensive industry. It requires abundant, cheap and skilled labour. Tea is
processed within the tea garden to restore its freshness.
● Major tea-producing states are Assam, hills of Darjeeling and Jalpaiguri districts, West
Bengal, Tamil Nadu and Kerala.
● Apart from these, Himachal Pradesh, Uttarakhand, Meghalaya, Andhra Pradesh and
Tripura are also tea-producing states in the country.
● Munnar in Kerala is one of the leading tea producing regions in India.

80. Answer: (c)


Explanation:
Rubber: It is an equatorial crop, but under special conditions, it is also grown in the tropical
and sub-tropical areas. It requires a moist and humid climate with rainfall of more than 200
cm., and temperature above 25°C.
Rubber is an important industrial raw material. It is mainly grown in Kerala, Tamil Nadu,
Karnataka and the Andaman & Nicobar Islands and the Garo hills of Meghalaya.

81. Answer: (b)


Explanation:
Cotton:
● India ranks second in the world in the production of cotton, after China.
● It is a kharif crop and requires 6 to 8 months to mature.
● Cotton, a semi-xerophyte, is grown in the tropical and sub-tropical conditions.
● Cotton is semi-tolerant to salinity and sensitive to water logging and thus, prefers well-
drained soils.

● Cotton grows well in the drier parts of the black cotton soil of the Deccan Plateau. It
requires high temperature, light rainfall or irrigation, 210 frost-free days and bright sun-
shine for its growth.
● Major cotton-producing states are Maharashtra, Gujarat, Madhya Pradesh, Karnataka,
Andhra Pradesh, Telangana, Tamil Nadu, Punjab, Haryana and Uttar Pradesh.

82. Answer: (a)


Explanation:
The Bhakra Nangal Project: It is the largest and the most important multi-purpose project,
named after the two dams built at Bhakra and Nangal on the Sutlej river. It is a joint
venture of Punjab, Haryana and Rajasthan states, designed to harness the precious
water of the Sutlej for the benefit of the concerned states.
The Bhakra Dam is one of the highest straight gravity dams in the world. It has been
constructed on the Sutlej river at the site of the Bhakra gorge near Rupnagar (Ropar),
in Himachal Pradesh and Punjab border. The dam has created a huge reservoir of water,
which is 88 km long and 8 km wide, with 2 storage capacities of 986.8 crore cubic metres.
This reservoir is named as the Gobind Sagar Lake, after Guru Gobind Singh, the tenth
Guru of the Sikhs.
The Nangal Hydel Channel is one of the longest cemented canals of the world. It takes off
from the left bank of the river and flows through rugged topography of steep slope. Its main
function is to turn the turbines of power houses located below the Nangal Dam, but it also
supplies water to the Bhakra irrigation canals. Power houses have been built to generate
hydroelectricity from water of the Sutlej river.
The reservoir of the dam, Gobind Sagar, homes fish of different species, including the
endangered ‘Mahseer’. Commercial fishing by local fisherman is also common in Gobind
Sagar. The Central Inland Fisheries Research Institute (CIFRI) has tracked the big drop in the
Bhakra Dam reservoir's fish population to a rise in the silt level because of muck dumping.

83. Answer: (b)


Explanation:
The Damodar Valley Corporation (DVC) is a government-owned power generator, which
operates in the Damodar river area of West Bengal and Jharkhand states of India, to handle
the Damodar Valley Project, the first multi-purpose river valley project of independent India.
The valley of the Damodar river was flood-prone and the devastating flood of 1943 lead to the
formation of the high-powered "Damodar Flood Enquiry Committee" by the government of
Bengal. The Damodar Valley Corporation was set up in 1948 as “the first multi-purpose river
valley project of independent India.”
The first dam was built across the Barakar river at Tilaiya and inaugurated in 1953.
The Konar Dam is the second of the four multi-purpose dams included in the first
phase of the Damodar Valley Corporation. It was constructed across the Konar river, a
tributary of the Damodar river in Hazaribagh district in the Indian state of Jharkhand,
and opened in 1955. The third dam across the Barakar river at Maithon was inaugurated in
1957. The fourth dam across the Damodar at Panchet was inaugurated in 1959.
Command Area: It is spread across the Damodar basin.
 Jharkhand: Dhanbad, Bokaro, Hazaribagh, Koderma, Chatra, Ramgarh, Palamau, Ranchi,
Lohardaga, Giridih and Dumka.
 West Bengal: Purba Bardhhaman, Paschim Bardhhaman, Hooghly, Howrah, Bankura and
Purulia.

84. Answer: (c)


Explanation:
The Hirakud Dam is built across the Mahanadi river, in the state of Odisha. It is the longest
earthen dam in the world. Behind the dam extends a lake, the Hirakud Reservoir. It is one of
the first major multi-purpose river valley projects started after India's independence. It also
forms the biggest artificial lake in India. The Hirakud Reservoir was declared a Ramsar
Site in 2021. The Hirakud Dam is a man-made structure of the earth, concrete and masonry.
In the upper drainage basin of the Mahanadi river, centred on the Chhattisgarh Plain,
periodic droughts contrast with the situation in the lower delta region, where floods may
damage the crops. The dam was constructed to help alleviate these problems, by creating a
reservoir and controlling river flow through the drainage system. The dam regulates the flow
of the Mahanadi river and produces hydroelectricity through several hydroelectric plants.
The dam with the channel provides an ideal environment for the wildlife. The Debrigarh
Wildlife Sanctuary is located here. Several species of migratory birds visit the reservoir during
winter.
● Cotton grows well in the drier parts of the black cotton soil of the Deccan Plateau. It
requires high temperature, light rainfall or irrigation, 210 frost-free days and bright sun-
shine for its growth.
● Major cotton-producing states are Maharashtra, Gujarat, Madhya Pradesh, Karnataka,
Andhra Pradesh, Telangana, Tamil Nadu, Punjab, Haryana and Uttar Pradesh.

85. Answer: (a)


Explanation:
The Chambal river is a tributary of the Yamuna river in Central and Northern India, and
thus, forms part of the drainage system of the Ganges. It is a legendary river and finds
mention in the ancient scriptures. The perennial Chambal originates south of Mhow town,
near Manpur, Indore, on the south slope of the Vindhyan Range in Madhya Pradesh. The
Chambal and its tributaries drain the Malwa region of north-western Madhya Pradesh,
while its tributary, the Banas, which rises in the Aravalli Range, drains south-eastern
Rajasthan.
The Chambal river is used for hydropower generation at the Gandhi Sagar Dam; Rana
Pratap Sagar Dam; and Jawahar Sagar Dam, and for annual irrigation in the
commands of the right main canal and the left main canal of the Kota Barrage.
The Gandhi Sagar Dam is the first of the four dams built on the Chambal river, located on
the Rajasthan-Madhya Pradesh border.
The Rana Pratap Sagar Dam is a dam located across the Chambal river near Rawatbhata in
Chittorgarh district in Rajasthan.
The Jawahar Sagar Dam is the third dam in the series of the Chambal Valley Projects,
located upstream of Kota city. The Kota Barrage is the fourth in the series of the Chambal
Valley Projects, located upstream of Kota city in Rajasthan. Water released after power
generation at the Gandhi Sagar Dam, Rana Pratap Sagar Dam and Jawahar Sagar Dam, is
diverted by the Kota Barrage for irrigation in Rajasthan and in Madhya Pradesh through
canals on the left and the right sides of the river.

86. Answer: (c)


Explanation:
The Rihand Dam, also known as Govind Ballabh Pant Sagar, is the second largest dam of
India by volume (storage), next only to Indira Sagar of Madhya Pradesh. The reservoir of
Rihand Dam is called Govind Ballabh Pant Sagar and is India's largest artificial lake. The
Rihand Dam is a concrete gravity dam located at Pipri in Sonbhadra district in Uttar
Pradesh. Its reservoir area is on the border of Madhya Pradesh and Uttar Pradesh. It is
located on the Rihand river, a tributary of the Son river. The catchment area of this dam
extends over Uttar Pradesh, Madhya Pradesh and Chhattisgarh, whereas it supplies
irrigation water in Bihar, located downstream of the river.

87. Answer: (b)


Explanation:
The Sardar Sarovar Project: The Sardar Sarovar Dam is a concrete gravity dam built on the
Narmada river in the state of Gujarat. The dam was constructed to provide water and
electricity to four Indian states: Gujarat, Madhya Pradesh, Maharashtra and Rajasthan.
The Gandak Project: The Gandak Project is a joint project of Bihar, Uttar Pradesh and
Nepal. The dam has been built at Bhaisalotan (Valmikinagar) in Bihar. An agreement was
signed between the Government of Nepal and the Government of India on the Gandak
Irrigation and Power Project.
The Idukki Project: The Idukki Dam is a double curvature arch dam constructed across the
Periyar river in a narrow gorge between two granite hills, locally known as Kuravan and
Kurathi in Idukki, Kerala.
The Bheema Project: The Bheema Lift Irrigation Project or the Rajiv Bheema Project, is
a lift irrigation canal project located in Telangana.
River Bhima, originating from the Western Ghats at Bhima Shankar near Karjat village in
Maharashtra state, is a tributary of the Krishna river.

88. Answer: (b)


Explanation:
The Indravati is an important tributary of river Godavari and contributes to about 20% of the
waters of the Godavari. It is the largest and the most important river in Bastar district of
Odisha.
The Nagarjuna Sagar Dam is a masonry dam across the Krishna river at Nagarjuna
Sagar, which straddles the border between Palnadu district in Andhra Pradesh and Nalgonda
district in Telangana.
The Nagarjuna Sagar Dam was the earliest in a series of large infrastructure projects, termed
as "modern temples" initiated for achieving the Green Revolution in India.
The construction of the dam submerged an ancient Buddhist settlement, Nagarjunakonda,
which was the capital of the Ikshvaku dynasty in the 1st and the 2nd centuries, and the
successors of the Satavahanas in the Eastern Deccan. Excavations yielded 30 Buddhist
monasteries, as well as artwork and inscriptions of historical importance.
The artificial lift irrigation based diversion of the river from its natural delta area into
Nalgonda district caused erosion of the fluorine-rich volcanic rocks in Nalgonda and
contaminated its groundwater supply. It also caused uncertain flows of water into the
Krishna river delta area and a shrinkage of the natural wonder, "The Kolleru Lake".

89. Answer: (d)


Explanation:
The Tehri Dam in Tehri Garhwal was conceived in 1949. Situated at the confluence of the
Bhagirathi and the Bhilganga rivers in Garhwal district of Uttarakhand, it is the highest in
Asia.
It is a multi-purpose rock and earth-fill embankment dam on the Bhagirathi river in New
Tehri, Tehri Garhwal district in Uttarakhand.
The Koteshwar Dam downstream creates the lower reservoir for the dam’s pumped-storage
plant. The dam helps in distributing power to the states/UTs of Uttarakhand, Rajasthan,
Punjab, Haryana, Uttar Pradesh, Jammu & Kashmir, Himachal Pradesh and Chandigarh.
The Tehri Dam has been the object of protests by the environmental organizations and local
people of the region. Environmental activist, Sunderlal Bahuguna, led the Anti-Tehri Dam
movement from 1980s till 2004. The protest was against the displacement of town
inhabitants and environmental consequence of the weak ecosystem. The Tehri Dam is in the
Central Himalayan Seismic Gap, a major geologic fault zone.

90. Answer: (b)


Explanation:
A joint venture of Punjab, Haryana and Rajasthan, the Beas Project consists of: (1) The
Beas-Sutlej Link; and (ii) The Pong Dam on the Beas river. The Beas-Sutlej Link involves the
construction of a diversion dam at Pandoh in Himachal Pradesh. The second unit includes a
dam at Pong in the Dhaoladhar Range near Pong village. It is mainly an irrigation scheme,
which is intended to ensure extension of perennial irrigation to hectares in Punjab, Haryana
and Rajasthan.
The Pong Dam, also known as the Beas Dam, is an earth-fill embankment dam on the Beas
river in the state of Himachal Pradesh, just upstream of Talwara. The purpose of the dam is
water storage for irrigation and hydroelectric power generation. The lake created by the
dam, Maharana Pratap Sagar, became a renowned bird sanctuary.
Maharana Pratap Sagar, also known as Pong Reservoir or Pong Dam Lake, is a large reservoir
in Kangra district of the state of Himachal Pradesh.
It was created in 1975, by building the highest earth-fill dam in India on the Beas river in the
wetland zone of the Siwalik Hills. Named in the honour of Maharana Pratap, the reservoir or
the lake is a well-known wildlife sanctuary and one of the 49 international wetland sites
declared in India by the Ramsar Convention.
The Pong Reservoir and the Gobind Sagar Reservoir are the two most important fishing
reservoirs in the Himalayan foothills of Himachal Pradesh.

91. Answer: (b)


Explanation:
Alluvial soils are depositional soils, transported and deposited by the rivers and the streams.
Through a narrow corridor in Rajasthan, they extend into the plains of Gujarat.
In the Peninsular region, they are found in the deltas on the east coast and in the river
valleys. The alluvial soils vary in nature from sandy loam to clay.
These soils cover about 40 percent of the country's total area. They are generally rich in
potash, but poor in phosphorous.
In the Upper and the Middle Ganga plains, two different types of alluvial soils have
developed, viz., Khadar and Bhangar. Khadar is the new alluvium and is deposited by floods
annually, which enriches the soil by depositing fine silts. Bhangar represents a system of
older alluvium, deposited away from the flood plains and it carries patches of alkaline
efflorescence, called usar, rendering some areas infertile.
In the Ganges basin, sandy aquifers, holding an enormous reserve of groundwater, ensure
irrigation and help make the plains the most agriculturally productive region of the country.
Both the Khadar and the Bhangar soils contain calcareous concretions (Kankars). These soils
are loamier and more clayey in the Lower and the Middle Ganga plains, and the
Brahmaputra valley. The sand content decreases from the west to the east.
The colour of the alluvial soils varies from light grey to ash grey. Its shades depend on the
depth of the deposition, the texture of the materials, and the time taken for attaining
maturity. Alluvial soils are intensively cultivated.

92. Answer: (b)


Explanation:
Black soil is an in-situ soil and it covers most of the Deccan Plateau, which includes parts of
Maharashtra, Madhya Pradesh, Gujarat, Andhra Pradesh and some parts of Tamil Nadu. In
the upper reaches of the Godavari and the Krishna, and the north-western part of the Deccan
Plateau, the black soil is very deep. These soils are also known as the ‘Regur Soil’ or the
‘Black Cotton Soil’.
The black soils are generally clayey, deep and impermeable. They swell and become sticky
when wet, and shrink when dried. So, during the dry season, these soil develop wide cracks.
Thus, there occurs a kind of ‘self-ploughing’. Because of this character of slow absorption
and loss of moisture, the black soil retains the moisture for a very long time, which helps the
crops, especially, the rain-fed ones, to sustain even during the dry season.
Chemically, the black soils are rich in lime, iron, magnesia and alumina. They also contain
potash. But they lack phosphorous, nitrogen and organic matter. The colour of the soil
ranges from deep black to grey.

93. Answer: (d)


Explanation:
Red soil develops on crystalline igneous rocks in the areas of low rainfall in the eastern and
southern parts of the Deccan Plateau. Along the piedmont zone of the Western Ghats, long
stretch of area is occupied by red loamy soil. Yellow and red soils are also found in parts of
Odisha and Chhattisgarh, and in the southern parts of the Middle Ganga plains. These are
also in-situ soils.
The soil develops a reddish colour due to a wide diffusion of iron in crystalline and
metamorphic rocks. It looks yellow when it occurs in a hydrated form.
The fine-grained red and yellow soils are normally fertile, whereas the coarse-grained soils,
found in the dry upland areas, are poor in fertility. They are generally poor in nitrogen,
phosphorous and humus.

94. Answer: (b)


Explanation:
The word ‘laterite’ has been derived from the Latin word ‘later’, which means brick. The
laterite soils develop in the areas with high temperature and high rainfall. These are the
result of intense leaching due to tropical rains. With rain, lime and silica are leached away,
and the soils, rich in iron oxide and aluminium compound, are left behind.
Humus content of the soil is removed fast by the bacteria that thrive well in high
temperature. These soils are poor in organic matter, nitrogen, phosphate and calcium, while
iron oxide and potash are in excess. Hence, laterites are not suitable for cultivation. However,
application of manures and fertilisers are required for making the soils fertile for cultivation.
Red laterite soils in Tamil Nadu, Andhra Pradesh and Kerala are more suitable for tree crops,
like cashewnut. Laterite soils are widely cut as bricks for use in house construction. These
soils have mainly developed in the higher areas of the Peninsular Plateau. The laterite soils
are commonly found in Karnataka, Kerala, Tamil Nadu, Madhya Pradesh and the hilly areas
of Odisha and Assam.

95. Answer: (c)


Explanation:
Arid soils range from red to brown in colour. They are generally sandy in structure and saline
in nature. In some areas, the salt content is so high that common salt is obtained by
evaporating the saline water. Due to the dry climate, high temperature and accelerated
evaporation, they lack moisture and humus. Nitrogen is insufficient and the phosphate
content is normal. Lower horizons of the soil are occupied by ‘Kankar’ layers, because of the
increasing calcium content downwards.
The ‘Kankar’ layer formation in the bottom horizons restricts the infiltration of water, and as
such, when irrigation is made available, the soil moisture is readily available for a
sustainable plant growth. Arid soils are characteristically developed in western Rajasthan,
which exhibits characteristic arid topography. These soils are poor, and contain little humus
and organic matter.

96. Answer: (c)


Explanation:
Saline soils are also known as Usara soils. Saline soils contain a larger proportion of sodium,
potassium and magnesium, and thus, they are infertile, and do not support any vegetative
growth.
They have more salts, largely because of dry climate and poor drainage. They occur in the
arid and semi-arid regions, and in the waterlogged and swampy areas. Their structure ranges
from sandy to loamy. They lack in nitrogen and calcium.
Saline soils are more widespread in western Gujarat, deltas of the eastern coast and in the
Sundarbans areas of West Bengal. In the Rann of Kuchchh, the south-west monsoon brings
salt particles and deposits there as a crust.
Seawater intrusions in the deltas promote the occurrence of saline soils. In the areas of
intensive cultivation with excessive use of irrigation, especially in the areas of the Green
Revolution, the fertile alluvial soils are becoming saline.
Excessive irrigation with dry climatic conditions promotes capillary action, which results in
the deposition of salt on the top layer of the soil. In such areas, especially in Punjab and
Haryana, the farmers are advised to add gypsum to solve the problem of salinity in the soil.

97. Answer: (c)


Explanation:
The peaty soils are found in the areas of heavy rainfall and high humidity, where there is a
good growth of vegetation. Thus, large quantity of dead organic matter accumulates in these
areas, and this gives a rich humus and organic content to the soil.
Organic matter in these soils may go even upto 40-50 per cent. These soils are normally
heavy and black in colour. At many places, they are alkaline also. They occur widely in the
northern part of Bihar, southern part of Uttarakhand, and the coastal areas of West Bengal,
Odisha and Tamil Nadu.

98. Answer: (a)


Explanation:
As the name suggests, forest soils are formed in the forest areas, where sufficient rainfall is
available. The soils vary in structure and texture, depending on the mountain environment
where they are formed. They are loamy and silty on the valley sides and coarse-grained
in the upper slopes. In the snow-bound areas of the Himalayas, they experience denudation
and are acidic with low humus content. The soils found in the lower valleys are fertile.

99. Answer: (b)


Explanation:
Sheet erosion takes place on the level lands after a heavy shower and the soil removal is not
easily noticeable. But, it is harmful, since it removes the finer and more fertile top soil.
Gully erosion is common on steep slopes. Gullies deepen with rainfall, cut the agricultural
lands into small fragments and make them unfit for cultivation. A region, with a large
number of deep gullies or ravines, is called a badland topography. Ravines are
widespread in the Chambal basin. Besides this, they are also found in Tamil Nadu and West
Bengal.

100. Answer: (c)


Explanation:
Kamarajar Port Limited, formerly Ennore Port, is located on the Coromandel Coast.
Paradip Port is a natural, deep-water port on the East coast of India in Paradip, Odisha. It is
at the confluence of the Mahanadi river and the Bay of Bengal.

You might also like